Sunteți pe pagina 1din 107

Polity- Subject wise compilation of 60 Day Plan- 2016

Q.1) Consider the following statements with regard to Constitution of India and select the
incorrect statement/s from the codes given below:
1. Constitution authorises the Parliament to alter the areas or boundaries of the existing
states without the consent of concerned state legislature or union territory.
2. Constitution does not guarantee any state of its territorial integrity or continued
existence
3. Constitution declares that laws made for admission or establishment of new states can
be passed by a simple majority
Choose the appropriate code
a) 1 only
b) 2 only
c) 3 only
d) None
Q.1) Solution (d)
Constitution authorises the Parliament to form new states or alter the areas, boundaries
or names of the existing states without the consent of concerned state legislature or
union territory. In other words, Parliament can redraw the political map of India
according to its will. Hence, the territorial integrity or continued existence of any state is
not guaranteed by the Constitution.
Constitution (Article 4) itself declares that laws made for admission or establishment of
new states (under Article 2) and formation of new states and alteration of areas,
boundaries or names of existing states (under Articles 3) are not to be considered as
amendments of the Constitution under Article 368. This means that such laws can be
passed by a simple majority and by the ordinary legislative process.
Source: Refer Chapter 5: Union and its Territory, Indian Polity by Laxmikanth
Doubt: @IASbaba Q1 Statement 2: What about JnK?
Clarifications: It is mentioned in Laxmikanth that, the provisions of the Constitution
pertaining to the states are applicable to all the states (except Jammu and Kashmir) in the
same manner.
So when we dealing with Part I (Article 1-4), the provisions mentioned will be applicable to
all states (and it is assumed by all states, it means excluding Jammu and Kashmir)
Q.2) Which among the following are the features of Government of India Act of 1935?
1. It introduced responsible governments in provinces
2. It created a new office, Secretary of State for India
3. It provided for the establishment of a Reserve Bank of India
4. It provided for the establishment of Supreme Court, which was set up in 1937

www.iasbaba.com

Polity- Subject wise compilation of 60 Day Plan- 2016

Choose the correct code


a) 1 and 2
b) 1 and 3
c) 1, 2 and 4
d) 1, 3 and 4
Q.2) Solution (b)
GoI Act, 1935 introduced responsible governments in provinces, that is, the governor
was required to act with the advice of ministers responsible to the provincial legislature.
It was GoI Act, 1858 which created a new office, Secretary of State for India, who was
vested with complete authority and control over Indian administration.
GoI Act, 1935 provided for the establishment of a Reserve Bank of India to control the
currency and credit of the country.
GoI Act, 1935 provided for the establishment of Federal Court (not Supreme Court),
which was set up in 1937. It was Regulating Act of 1773 which provided for the
establishment of Supreme Court at Calcutta (1774)
Source: Refer Chapter 1: Historical Background, Indian Polity by Laxmikanth
Doubt: q-2 according to your answer sheet goi act 1935 provides for establishment of rbi
but this is wrong rbi was estd through rbi act of 1934. check it out https://www.rbi.org.in/Scripts/AboutusDisplay.aspx#EP
Clarification: Statement 3 was picked directly from the source mentioned above (under
features of GoI Act, 1935)
*It is advised to stick to Laxmikanth, dont go deep to study individual documents and waste
time.
Even when we read The Reserve Bank of India Act, 1934 from your link, Chapter 1 page
13 sub-clause (3) we come across [.provisions of this Act shall come into force on such
date or dates3 as the 4 [Central Government] may, by notification in the Gazette of India,
appoint.]
Suffix 3 in that document says All the sections by 1st April 1935
So Central Government (through GoI Act, 1935) might have provided for the establishment
of a Reserve Bank of India.

Q.3) According to Dr B R Ambedkar, which among the following is a novel feature of the
Indian Constitution?
a) Directive Principles of State Policy
b) Fundamental Rights
c) Preamble

www.iasbaba.com

Polity- Subject wise compilation of 60 Day Plan- 2016


d) Judicial Review
Q.3) Solution (a)
According to Dr B R Ambedkar, the Directive Principles of State Policy is a novel feature
of the Indian Constitution.
Dr. Ambedkar said The directive principles are like instruments of instructions which
were issued to the Governor in General and Governors of colonies and to those of India
by the British Government under the 1935 Act under the Draft Constitution. It is
proposed to issue such instructions to the president and governors. The text of these
instruments of the instructions shall be found in scheduled IV to the Constitution of
India. What are called directive principles is that they are instructions to the Legislature
and the Executive. Such a thing is, to my mind, to be welcomed.
The Directive Principles commit the State to promote the welfare of the people by
affirming social, economic and political justice, as well as to fight economic inequality.
Source: Refer Chapter 3: Salient Features of the Constitution, Indian Polity by Laxmikanth
Q.4) In which of the following points is the Indian Constitution similar to that of Canadian
Constitution?
1. Federation with a strong Centre
2. Office of Governor
3. Advisory jurisdiction of the Supreme Court
4. Vesting of residuary powers in the Centre
Choose the correct codes
a) 1, 2 and 3
b) 2, 3 and 4
c) 1, 3 and 4
d) All of the above
Q.4) Solution (c)
The following are the features borrowed by Indian Constitution from the Canadian
Constitution
1) Federation with a strong Centre
2) Vesting of residuary powers in the Centre
3) Appointment of state governors by the Centre (not Office of Governor)
4) Advisory jurisdiction of the Supreme Court
Office of Governor was borrowed from Government of India Act of 1935
Source: Refer Chapter 3: Salient Features of the Constitution, Indian Polity by Laxmikanth

www.iasbaba.com

Polity- Subject wise compilation of 60 Day Plan- 2016


Q.5) Which one of the following Acts of British India established a system of double
government?
a) Government of India Act of 1919
b) Indian Councils Act of 1909
c) Government of India Act of 1935
d) Pitts India Act of 1784
Q.5) Solution (d)
The Pitts India Act of 1784 distinguished between the commercial and political
functions of the Company.
It allowed the Court of Directors to manage the commercial affairs but created a new
body called Board of Control to manage the political affairs. Thus, it established a system
of double government.
It empowered the Board of Control to supervise and direct all operations of the civil and
military government or revenues of the British possessions in India.
Source: Refer Chapter 1: Historical Background, Indian Polity by Laxmikanth
Q.6) Consider the following statements with regard to States and Union Territories. Which
of the following statement is/are incorrect?
1. Union of India is a wider expression than the Territory of India.
2. The states are the members of the federal system and share a distribution of powers
with the Centre.
3. The union territories and the acquired territories, on the other hand, are directly
administered by the Central government.
Choose the correct codes
a) 1 only
b) 3 only
c) 2 and 3 only
d) None
Q.6) Solution (a)
Territory of India is a wider expression than the Union of India because the latter
includes only states while the former includes not only the states but also union
territories and territories that may be acquired by the Government of India at any future
time.
The states are the members of the federal system and share a distribution of powers
with the Centre.
The union territories and the acquired territories, on the other hand, are directly
administered by the Central government.

www.iasbaba.com

Polity- Subject wise compilation of 60 Day Plan- 2016

Source: Refer Chapter 5: Union and its Territory, Indian Polity by Laxmikanth
Q.7) In which case, the Supreme Court held that the Indian Constitution is founded on the
bedrock of the balance between the Fundamental Rights and the Directive Principles?
a) Kesavananda Bharati case
b) Berubari case
c) Golaknath case
d) Minerva Mills case
Q.7) Solution (d)
It was in the Minerva Mills case (1980), the Supreme Court held that the Indian
Constitution is founded on the bedrock of the balance between the Fundamental Rights
and the Directive Principles.
Source: Refer Chapter 3: Salient Features of the Constitution, Indian Polity by Laxmikanth
Q.8) Which Schedule of the Constitution deals with Acts and Regulations of the state
legislatures dealing with land reforms and abolition of the zamindari system?
a) Schedule Eight
b) Schedule Nine
c) Schedule Eleventh
d) Schedule Twelfth
Q.8) Solution (b)
Ninth Schedule deals with Acts and Regulations of the state legislatures dealing with
land reforms and abolition of the zamindari system and of the Parliament dealing with
other matters.
This schedule was added by the 1st Amendment (1951) to protect the laws included in it
from judicial scrutiny on the ground of violation of fundamental rights.
However, in 2007, the Supreme Court ruled that the laws included in this schedule after
April 24, 1973, are now open to judicial review.
Source: Refer Chapter 3: Salient Features of the Constitution, Indian Polity by Laxmikanth
Q.9) Which one of the following Acts of British India introduced for the first time
bicameralism and direct elections in the country?
a) Indian Council Act of 1892
b) Government of India Act of 1919
c) Government of India Act of 1935
d) Indian Independence Act of 1947

www.iasbaba.com

Polity- Subject wise compilation of 60 Day Plan- 2016

Q.9) Solution (b)


Government of India Act of 1919 which is also known as Montagu-Chelmsford Reforms
introduced for the first time bicameralism and direct elections in the country.
Thus, the Indian Legislative Council was replaced by a bicameral legislature consisting of
an Upper House (Council of State) and a Lower House (Legislative Assembly).
The majority of members of both the Houses were chosen by direct election.
Source: Refer Chapter 1: Historical Background, Indian Polity by Laxmikanth
Q.10) Match the following
Type of Writ
1. Habeas Corpus
2. Certiorari
3. Mandamus
4. Quo-Warranto
Select the correct code
a) 1-A, 2-B, 3-C, 4-D
b) 1-C, 2-A, 3-B, 4-D
c) 1-C, 2-B, 3-A, 4-D
d) 1-D, 2-B, 3-C, 4-A

Meaning
A. we command
B. to be certified
C. you may have the body
D. what is your authority

Q.10) Solution (c)


Types of Writs
There are five types of writs in the Indian Constitution - Habeas Corpus, Certiorari, QuoWarranto, Mandamus and Prohibition.
1. Writ of Habeas Corpus This writ literally means you may have the body. This writ is issued to produce a person
physically before the court who has been imprisoned or detained by the law and to set him
free if there is no legal justification of his detention.
2. Writ of Certiorari
This writ literally means 'to be certified. This writ is issued by the higher court to the lower
court for quashing the order already passed by the lower court or removing a suit from the
lower court to higher court for speedy disposal.
3. Writ of Quo- Warranto
This writ literally means by what warrants? or what is your authority. It is a writ issued by
the Supreme Court or High Court with a view to restrain a person or authority from holding
a public office to which he is not entitled. The writ requires the concerned person or
authority to explain to the Court by what authority he/it holds the office.
4. Writ of Mandamus

www.iasbaba.com

Polity- Subject wise compilation of 60 Day Plan- 2016


This writ literally means we command. It is a a judicial writ issued by the Supreme Court or
a High Court as a command to an interior court or tribunal or a person to perform a public
or statutory duty.
5. Writ of Prohibition
This writ is popularly known as Stay Order. This writ is issued by a higher court to a lower
court to stop proceeding in a case on the ground of over-stepping of jurisdiction or absence
of jurisdiction. It is issued before the judgement or order is made in such cases.
Source: Refer Chapter 3 and Chapter 7: Salient Features of the Constitution and
Fundamental Rights, Indian Polity by Laxmikanth
Q.11) Which one of the following Acts of British India strengthened the Viceroys authority
over his executive council by substituting Portfolio or departmental system for corporate
functioning?
a) Indian Council Act, 1861
b) Government of India Act, 1858
c) Indian Council Act, 1892
d) Indian Council Act, 1909
Q.11) Solution (a)
Indian Council Act, 1861 empowered the Viceroy to make rules and orders for the more
convenient transaction of business in the council.
It also gave a recognition to the portfolio system, introduced by Lord Canning in 1859.
Under this, a member of the Viceroys council was made in-charge of one or more
departments of the government and was authorised to issue final orders on behalf of
the council on matters of his department(s).
Source: Refer Chapter 1: Historical Background, Indian Polity by Laxmikanth
Q.12) Consider the following statements with regard to the Constitution
1. The Constitution prohibits discrimination against any citizen on grounds of religion, race,
caste, sex or place of birth but not on the ground of residence.
2. The Constitution of India has introduced the system of single citizenship and provided
uniform rights for the people of India to promote the feeling of fraternity and unity
among them and to build an integrated Indian nation.
Which among the above statements is/are true?
a) 1 only
b) 2only
c) Both 1 and 2
d) Nether 1 nor 2

www.iasbaba.com

Polity- Subject wise compilation of 60 Day Plan- 2016


Q.12) Solution (c)
The Constitution (under Article 15) prohibits discrimination against any citizen on
grounds of religion, race, caste, sex or place of birth and not on the ground of residence.
This means that the state can provide special benefits or give preference to its residents
in matters that do not come within the purview of the rights given by the Constitution to
the Indian citizens. For example, a state may offer concession in fees for education to its
residents.
The Constitution of India, like that of Canada, has introduced the system of single
citizenship and provided uniform rights (except in few cases) for the people of India to
promote the feeling of fraternity and unity among them and to build an integrated
Indian nation.
Despite this, India has been witnessing the communal riots, class conflicts, caste wars,
linguistic clashes and ethnic disputes. Thus, the cherished goal of the founding fathers
and the Constitution-makers to build an united and integrated Indian nation has not
been fully realized.
Source: Refer Chapter 6: Citizenship, Indian Polity by Laxmikanth
Q.13) The detailed provisions regarding acquisition and termination of Indian citizenship
are contained in the Citizenship Act which was passed by
a) The Indian Parliament in 1955
b) The Indian Parliament in 1950
c) The British Parliament in August 1948
d) The Constituent Assembly in 1949
Q.13) Solution (a)
The Constitution empowered the Parliament to enact a law to provide for acquisition
and loss of citizenship after the commencement of the Constitution.
Accordingly, the Parliament has enacted the Citizenship Act, 1955, which has been
amended in 1986, 1992, 2003 and 2005.
Source: Refer Chapter 6: Citizenship, Indian Polity by Laxmikanth
Q.14) Consider the statements regarding loss of citizenship:
1. If a person voluntarily renounces Indian citizenship, every minor child of that person also
loses his citizenship.
2. When a citizen voluntarily acquires the citizenship of another country, his Indian
citizenship automatically terminates. This provision does not apply during a war in which
India is engaged.
3. The citizenship is terminated if the citizen has unlawfully traded or communicated with
the enemy during a war.

www.iasbaba.com

Polity- Subject wise compilation of 60 Day Plan- 2016

Which of the above statements are correct?


a) 1 and 2
b) 2 and 3
c) 1 and 3
d) All of the above
Q.14) Solution (d)
If a person voluntarily renounces Indian citizenship, every minor child of that person also
loses his citizenship. However, when such a child attains the age of eighteen, he may
resume Indian citizenship
Refer Laxmikanth Loss of citizenship
Q.15) Consider the following statements regarding Constitutional provisions of
Citizenship:
1. The Part II of the Indian Constitution deals with Citizenship.
2. Detailed provisions of acquisition and loss of citizenship are mentioned in the
Constitution.
3. Any law made regarding citizenship is considered as a Constitutional Amendment.
Which of the above statements are incorrect?
a) 1 only
b) 2 and 3
c) 3 only
d) None of the above
Q.15) Solution (b)
The constitution neither contains any permanent nor elaborate provisions regarding
citizenship. It empowers the parliament to enact a law to deal with the problem of
acquisition or loss of citizenship. Laws made by the parliament regarding citizenship are not
considered as Constitutional amendment.
Q.16) Which of the statements regarding Overseas Citizen of India cardholder are correct?
1. They get a multiple entry, multipurpose lifelong visa for visiting India.
2. They get parity with NRIs in respect of economic, financial and education fields except
acquisition of agricultural lands.
3. They are required to register with local police authorities once their stay in India exceeds
180 days for the first time.
Select the correct code from the following:

www.iasbaba.com

Polity- Subject wise compilation of 60 Day Plan- 2016


a)
b)
c)
d)

1 and 2
2 and 3
1 and 3
All of the above

Q.16) Solution (a)


OCIs dont need to get registered with local police station.
Q.17) Article 13 declares that all laws that are inconsistent with or in derogation of any of
the fundamental rights shall be void. Which among the following laws can be challenged
in courts as violating FR?
1. Ordinances issued by the president or the state governors.
2. Delegated legislations.
3. Constitutional amendments.
Choose the correct codes
a) 1 and 2 only
b) 1 and 3 only
c) 3 only
d) All the above
Q.17) Solution (d)
The term law in Article 13 has been given a wide connotation so as to include the
following:
(a) Permanent laws enacted by the Parliament or the state legislatures;
(b) Temporary laws like ordinances issued by the president or the state governors;
(c) Statutory instruments in the nature of delegated legislation (executive legislation) like
order, bye-law, rule, regulation or notification; and
(d) Non-legislative sources of law, that is, custom or usage having the force of law.
Further, Article 13 declares that a constitutional amendment is not a law and hence cannot
be challenged. However, the Supreme Court held in the Kesavananda Bharati case2 (1973)
that a Constitutional amendment can be challenged on the ground that it violates a
fundamental right that forms a part of the basic structure of the Constitution and hence,
can be declared as void.
Q.18) The term State has been used in different provisions concerning the fundamental
rights. According to Article 12, the State includes which among the following entities
1. Executive organs of the Union and state governments.
2. Legislative organs of the Union and state governments.
3. Judiciary at Union and state levels.
4. Local Self-governments created under 73rd and 74th amendments.

www.iasbaba.com

10

Polity- Subject wise compilation of 60 Day Plan- 2016


5. Statutory authorities.
6. Non-statutory authorities.
Choose the correct codes
a) 1, 2, 3, 4 and 5 only
b) 1, 2, 4, 5 and 6 only
c) 1, 2, 3 and 4 only
d) All the above
Q. 18) Solution (b)
According to Article 12, the State includes the following:
(a) Government and Parliament of India, that is, executive and legislative organs of the
Union government.
(b) Government and legislature of states, that is, executive and legislative organs of state
government.
(c) All local authorities that is, municipalities, panchayats, district boards, improvement
trusts, etc.
(d) All other authorities, that is, statutory or non-statutory authorities like LIC, ONGC, SAIL,
etc.
Thus, State has been defined in a wider sense so as to include all its agencies. It is the
actions of these agencies that can be challenged in the courts as violating the Fundamental
Rights. According to the Supreme Court, even a private body or an agency working as an
instrument of the State falls within the meaning of the State under Article 12.
Q.19) Which among the following FR does a foreign citizen can avail
1. Freedom to manage religious affairs.
2. Freedom of conscience and free profession, practice and propagation of religion.
3. Freedom from attending religious instruction or worship in certain educational
institutions.
4. Right to elementary education.
5. Protection of language, script and culture of minorities.
6. Right of minorities to establish and administer educational institutions.
Choose the correct codes
a) 1, 2, 3, 4 and 5 only
b) 1, 2, 3, 5 and 6 only
c) 1, 2, 3 and 4 only
d) All the above
Q.19) Solution (c)
FR available to both citizens and foreigners (except enemy aliens):

www.iasbaba.com

11

Polity- Subject wise compilation of 60 Day Plan- 2016


1.
2.
3.
4.
5.
6.
7.
8.

Equality before law and equal protection of Laws (Article 14).


Protection in respect of conviction for offences (Article 20).
Protection of life and personal liberty (Article 21).
Right to elementary education (Article 21A).
Protection against arrest and detention in certain cases (Article 22).
Prohibition of traffic in human beings and forced labour (Article 23).
Prohibition of employment of children in factories etc., (Article 24).
Freedom of conscience and free profession, practice and propagation of religion (Article
25).
9. Freedom to manage religious affairs (Article 26).
10. Freedom from payment of taxes for promotion of any religion (Article 27).
11. Freedom from attending religious instruction or worship in certain educational
institutions (Article 28).
Q.20) The concept "Equality before law" connotes which among the following
1. The absence of any special privileges in favour of any person.
2. The equality of treatment under equal circumstances.
3. The equal subjection of all persons to the ordinary law of the land.
Choose the correct codes
a) 1 and 2 only
b) 1 and 3 only
c) 2 and 3 only
d) All the above
Q.20) Solution (b)
The concept "Equality before law" connotes:
(a) The absence of any special privileges in favour of any person,
(b) The equal subjection of all persons to the ordinary law of the land administered by
ordinary law courts, and
(c) No person (whether rich or poor, high or low, official or non-official) is above the law.
On the other hand, "Equal protection of laws" connotes:
(a) The equality of treatment under equal circumstances, both in the privileges conferred
and liabilities imposed by the laws,
(b) The similar application of the same laws to all persons who are similarly situated, and
(c) The like should be treated alike without any discrimination.
Q.21) Consider the following statements regarding Fundamental Duties enshrined in the
constitution of India
1. These are applicable to all the residents of India irrespective of their caste, creed, sex
and religion.

www.iasbaba.com

12

Polity- Subject wise compilation of 60 Day Plan- 2016


2. The parliament cannot impose legal sanction against the violation of Fundamental
duties.
Choose the correct codes
a) 1 only
b) 2 only
c) Both 1 and 2
d) Neither 1 nor 2
Q.21) Solution (d)
Unlike some of the Fundamental Rights which extend to all persons whether citizens or
foreigners, the Fundamental Duties are confined to citizens only and do not extend to
foreigners. So, statement 1 is wrong as residents of India may include foreign citizens also.
Like the Directive Principles, the fundamental duties are also non-justiciable. The
Constitution does not provide for their direct enforcement by the courts. Moreover, there is
not legal sanction against their violation. However, the Parliament is free to enforce them
by suitable legislation. So, statement 2 is also wrong.
Q.22) Which of the following statements can be considered as significance of Fundamental
Duties?
1. They help the courts in examining and determining the constitutional validity of a law.
2. They are enforceable by law.
Choose the correct codes
a) 1 only
b) 2 only
c) Both 1 and 2
d) Neither 1 nor 2
Q.22) Solution (c)
They help the courts in examining and determining the constitutional validity of a law. In
1992, the Supreme Court ruled that in determining the constitutionality of any law, if a
court finds that the law in question seeks to give effect to a fundamental duty, it may
consider such law to be reasonable in relation to Article 14 (equality before law) or Article
19 (six freedoms) and thus save such law from unconstitutionality. Hence, statement 1 is
correct.
It is true that there is no legal sanction provided for violation or non-performance of
Fundamental Duties. There is neither specific provision for enforceability nor any specific
prohibition.

www.iasbaba.com

13

Polity- Subject wise compilation of 60 Day Plan- 2016


However, Fundamental Duties have an inherent element of compulsion regarding
compliance. Out of the ten clauses in article 51A, five are positive duties and the other five
are negative duties. Clauses (b), (d), (f), (h) and (j) require the citizens to perform these
Fundamental Duties actively.
It is said that by their nature, it is not practicable to enforce the Fundamental Duties and
they must be left to the will and aspiration of the citizens.
However, in the case of citizens holding public office, each and all Fundamental Duties can
be enforced by suitable legislation and departmental rules of conduct.
Appropriate sanctions can be provided for lapse in respect of each Fundamental Duty and it
is quite practicable to enforce the sanction against every citizen holding a public office; for
instance, departmental promotions can be deferred, increments can be withheld, etc. If an
officer takes part in a strike or stalls the proceedings of his institution, he can be made to
forgo the salary for that day.
There is a need for these duties to be obligatory for all citizens, subject to the State
enforcing the same by means of a valid law, or else the law stands in a very disadvantageous
position. The Supreme Court has finally, issued directions to the State in this regard, with a
view towards making the provisions effective and enabling a citizens to properly perform
their duties properly.

Q.23) Which among the following legal provisions are meant for the implementation of
some of the Fundamental Duties enshrined in the constitution of India?
1. Unlawful activities (Prevention) Act.
2. Representation of People Act (1951).
3. The Undisclosed Foreign Income and Assets (Imposition of Tax) Bill, 2015.
Choose the correct codes
a) 1 and 2 only
b) 1 and 3 only
c) 2 and 3 only
d) All the above
Q.23) Solution (a)
The Verma Committee on Fundamental Duties of the Citizens (1999) identified the existence
of legal provisions for the implementation of some of the Fundamental Duties.
The Unlawful Activities (Prevention) Act of 1967 provides for the declaration of a communal
organisation as an unlawful association. Hence, statement 1 is correct.

www.iasbaba.com

14

Polity- Subject wise compilation of 60 Day Plan- 2016


The Representation of People Act (1951) provides for the disqualification of members of the
Parliament or a state legislature for indulging in corrupt practice that is, soliciting votes on
the ground of religion or promoting enmity between different sections of people on grounds
of caste, race, language, religion and so on. So, statement 2 is also correct.
Duty to pay taxes is not considered as Fundamental Duty. Hence, statement 3 is wrong.
Q.24) Which among the following is called as "Conscience of the constitution" by Granville
Austin?
1. Fundamental Rights.
2. Fundamental Duties.
3. Directive Principles of State Policy
Choose the correct codes
a) 1 and 2 only
b) 1 only
c) 1 and 3 only
d) All the above
Q.24) Solution (c)
The Directive Principles along with the Fundamental Rights contain the philosophy of the
Constitution and is the soul of the Constitution. Granville Austin has described the Directive
Principles and the Fundamental Rights as the Conscience of the Constitution
Q.25) DPSP are like the instrument of instructions. These instructions or recommendations
to the State in which among the following matters
1. Legislative organs of the central and state governments.
2. Executive organs of the central and state governments.
3. Judiciary at Union and state levels.
4. All local authorities of the country.
5. All public authorities of the country.
Choose the correct codes
a) 1, 2, 3 and 4 only
b) 1, 2, 4 and 5 only
c) 1, 2 and 3 only
d) All the above
Q. 25) Solution (b)
Directive Principles of State Policy are the constitutional instructions or recommendations
to the State in legislative, executive and administrative matters. According to Article 36, the
term State in Part IV has the same meaning as in Part III dealing with Fundamental Rights.

www.iasbaba.com

15

Polity- Subject wise compilation of 60 Day Plan- 2016


Therefore, it includes the legislative and executive organs of the central and state
governments, all local authorities and all other public authorities in the country.
Q.26) In Minerva mills case (1980), according to Supreme Court which among the
following constitutes the core of commitment to social revolution
1. Fundamental Rights.
2. Fundamental Duties.
3. Directive Principles of State Policy
Choose the correct codes
a) 1 and 2 only
b) 1 only
c) 1 and 3 only
d) All the above
Q.26) Solution (c)
In the Minerva Mills case (1980), the Supreme Court also held that the Indian Constitution is
founded on the bedrock of the balance between the Fundamental Rights and the Directive
Principles. They together constitute the core of commitment to social revolution.
Q.27) Consider the following statements regarding Quo-Warranto:
1. It prevents illegal usurpation of public office by a person.
2. It cannot be issued in case of Ministerial office or private office.
3. This can be sought by any interested person and not necessarily by the aggrieved
person.
Which of the above statements are correct?
a) 1 and 2
b) 1 and 3
c) 2 and 3
d) All of the above
Q.27) Solution (d)
Refer Laxmikanth Fundamental Rights, Quo-Waranto.
Q.28) Article 34 of the Indian Constitution provides for the restriction on fundamental
rights while Martial Law is in force in any area within the territory of India. Which of the
following statements regarding Martial Law are correct?
1. The concept of Martial law has been borrowed from the English Common Law.
2. It refers to a situation where civil administration is run by military authorities according
to their own rules and regulations.

www.iasbaba.com

16

Polity- Subject wise compilation of 60 Day Plan- 2016


3. The expression Martial Law has not been defined anywhere in the constitution.
Select the correct code from the following:
a) 1 and 2
b) 2 and 3
c) 1 and 3
d) All of the above
Q.28) Solution (d)
All the statements are correct. During the operation of martial law, the military authorities
are vested with abnormal powers to take all necessary. They impose restrictions and
regulations on the rights of the civilians, can punish the civilians and can condemn them to
death.
Q.29) Which of the following Fundamental Rights are available only to citizens and not to
foreigners?
1. Prohibition of discrimination on grounds of religion, race, caste, sex or place of birth.
2. Equal opportunity in matters of public employment.
3. Freedom of speech and expression.
4. Protection of language, script and culture of minorities.
Select the correct code from the following:
a) 4 only
b) 2,3 and 4
c) 1,2 and 3
d) All of the above
Q.29) Solution (d)
FRs available only to citizens and not to foreigners are:
Article 15- prohibition of discrimination on the basis of religion, race, caste, sex or place of
birth.
Article 16- Equality of opportunity in matters of public employment.
Article 19- Six freedoms.
Article 29- Protection of language, script and culture of minorities.
Article 30- Right of minorities to establish and administer educational institutions.
Q.30) Consider the following statements regarding Uniform Civil Code:
1. It is a Directive Principle of State Policy given under article 45 of Indian Constitution.
2. It is a Gandhian directive principle.
3. Uniform Civil Code refers to the body of laws governing rights and duties pertaining to
property and personal matters like marriage, divorce, adoption and inheritance.

www.iasbaba.com

17

Polity- Subject wise compilation of 60 Day Plan- 2016

Which of the above statements are correct?


a) All of the above
b) 2 and 3
c) 1 and 2
d) 3 only
Q.30) Solution (d)
It is a DPSP given in article-44
It is a Liberal- Intellectual principle.
Q.31) Which of the following statements are a correct comparison between Fundamental
Rights(FR) and Directive Principle of State Policy (DPSPs):
1. FRs are justiciable while DPSPs are not.
2. FRs are negative as they prohibit state from doing certain things while DPSPs are
positive as they require state to do certain things.
3. In a conflict between DPSPs and FRs, FRs always prevails.
Select the correct code from the following:
a) 1 and 2
b) 2 and 3
c) 1 and 3
d) All of the above
Q.31) Solution (a)
Directive Principles given in Article 39 (b) and (c) have been given superiority over Article 14
and Article 19.
Q.32) Consider the following regarding the Fundamental Duties:
1. They were added as Part IVA of the Constitution by 44th Amendment Act.
2. Initially there were only 10 Fundamental Duties.
3. 86th Constitution Amendment Act 2002, added the duty to provide education to ones
child or ward between the age of 6 and 14 years.
Which of the above statements are incorrect?
a) 2 and 3
b) 1 only
c) 1 and 2
d) None of the above
Q.32) Solution (b)

www.iasbaba.com

18

Polity- Subject wise compilation of 60 Day Plan- 2016


Fundamental Duties were added by 42nd Constitutional Amendment Act.
Q.33) Which of the following statements is/are Constitutional Provision for Amendment
of the Indian Constitution?
1) Parliament has been bestowed with the constituent power, using which changes in the
Indian Constitution can be made
2) It is mentioned in Constitution that there will be no limitation on the constituent power
of the Parliament for amending it by adding, removing or improving the provisions made
in it
Choose the appropriate code:
a) 1 only
b) 2 only
c) 1 and 2 only
d) None of the above
Q.33) Solution (c)
Explanation:
Article 368 in Part XX of the Constitution deals with the powers of Parliament to amend
the Constitution and its procedure.
It states that the Parliament may, in exercise of its constituent power, amend by way of
addition, variation or repeal any provision of the Constitution in accordance with the
procedure laid down for the purpose.
For the removal of doubts, it is hereby declared that there shall be no limitation
whatever on the constituent power of Parliament to amend by way of addition,
variation or repeal the provisions of this Constitution under this article.
However, the Supreme Court in Minerva Mills Ltd. and Others Vs. Union of India and
Others (1980), declared the above section as invalid and unconstitutional.
(Refer the note given under clarifications below)
Source: Chapter 10 - Amendment of the Constitution, Laxmikanth
Doubts: Q11- limited amending power is one of the basic features of the Constitution...then
HW can Constitution confer unlimited power on Parliament..?? 2nd option may be wrong.
Can someone explain Q11? There is a limit that parliament can not repeal FR or any privision
which are basic structure(keshavnanda bharti case) of the constitution
The given statement (no limitation) is wrong.......as basic structure cannot be
amended........please clarify IASbaba
Clarification:

www.iasbaba.com

19

Polity- Subject wise compilation of 60 Day Plan- 2016


The following is the full text of Article 368 of the Constitution, which governs constitutional
amendments.
Link: http://lawmin.nic.in/olwing/coi/coi-english/Const.Pock%202Pg.Rom8Fsss(26).pdf
* [(4) No amendment of this Constitution (including the provisions of Part III) made or
purporting to have been made under this article whether before or after the
commencement of section 55 of the Constitution (Forty-second Amendment) Act, 1976]
shall be called in question in any court on any ground.
(5) For the removal of doubts, it is hereby declared that there shall be no limitation
whatever on the constituent power of Parliament to amend by way of addition, variation or
repeal the provisions of this Constitution under this article.]
*This section [Cls. (4) and (5)] has been declared invalid (or we can say unconstitutional) by
the Supreme Court in Minerva Mills Ltd. and Others Vs. Union of India and Others (1980)

Note: Very important to remember this


The Constitution can be amended only by Parliament; and only in the manner provided.
Although Parliament must preserve the basic framework of the Constitution, there is no
other limitation placed upon the amending power, meaning that there is no provision of
the Constitution that cannot be amended.
Our intention was to grow a seed of doubt in your minds. So you learn this above concept
and do not forget.
If you come across any question, for instance, a statement asking that there is no provision
of the Constitution that cannot be amended from the above note, now it is easy to
answer it is true.
There is no limitation (except Parliament must preserve the basic framework of the
Constitution). Hence, there is a minor limitation.
In other words, it is mentioned in the constitution that the no limitation provision was
made void or unconstitutional by the Supreme Court (Minerva Mills Ltd. and Others Vs.
Union of India and Others (1980))

Q. 34) Which among the following takes place during election and impeachment of
President of India?

www.iasbaba.com

20

Polity- Subject wise compilation of 60 Day Plan- 2016


1) The elected members of the legislative assemblies and councils of the states and UTs of
Delhi and Puducherry participate in election but do not participate in impeachment
2) The nominated members of either House of Parliament do not participate in election but
participate in impeachment
Choose the appropriate code:
a) 1 only
b) 2 only
c) 1 and 2 only
d) None of the above
Q.34) Solution (b)
Explanation:
The elected members of the legislative assemblies of the states and UTs of Delhi and
Puducherry participate in election but do not participate in impeachment (no legislative
councils)
The nominated members of either House of Parliament do not participate in election but

Source: Chapter 17 - President, Laxmikanth


Q. 35) Consider the following with regard to Ordinance making power of President:
1) Ordinance making is the discretionary power of the President of India
2) President can promulgate or withdraw an ordinance only on the advice of Cabinet
Ministers
Which of the statements given above is/are incorrect?
a) 1 only
b) 2 only
c) Both 1 and 2
d) Neither 1 nor 2
Q.35) Solution (c)
Explanation:
Article 123 of the Constitution empowers the President to promulgate ordinances during
the recess of Parliament. These ordinances have the same force and effect as an act of
Parliament, but are in the nature of temporary laws.
He can promulgate an ordinance only when both the Houses of Parliament are not in
session or when either of the two Houses of Parliament is not in session. An ordinance
can also be issued when only one House is in session because a law can be passed by
both the Houses and not by one House alone. An ordinance made when both the

www.iasbaba.com

21

Polity- Subject wise compilation of 60 Day Plan- 2016

Houses are in session is void. Thus, the power of the President to legislate by ordinance
is not a parallel power of legislation.
However, his power of ordinance-making is not a discretionary power, and he can
promulgate or withdraw an ordinance only on the advice of the council of ministers
headed by the prime minister. (not Cabinet Ministers)

Source: Chapter 17 - President, Laxmikanth


Q. 36) Which among the following are correct with regard to Council of Ministers and
Cabinet?
1) Council of Ministers does not meet, as a body, to transact government business. It has
no collective functions.
2) Cabinet meets, as a body, frequently and usually once in a week to deliberate and take
decisions regarding the transaction of government business. Thus, it has collective
functions.
3) Council of Ministers enforces the collective responsibility of the cabinet to the Lower
House of Parliament.
4) Cabinet is collectively responsible to the Lower House of the Parliament.
Choose the correct code from below options:
a) 1 and 2 only
b) 2 and 4 only
c) 3 and 4 only
d) All of the above
Q.36) Solution (a)
Explanation:
Council of Ministers does not meet, as a body, to transact government business. It has
no collective functions.
Cabinet meets, as a body, frequently and usually once in a week to deliberate and take
decisions regarding the transaction of government business. Thus, it has collective
functions.
Cabinet enforces the collective responsibility of the council of ministers to the Lower
House of Parliament.
Council of Ministers is collectively responsible to the Lower House of the Parliament.
Chapter 20 Central Council of Ministers, Laxmikanth
Q. 37) Consider the following statements:

www.iasbaba.com

22

Polity- Subject wise compilation of 60 Day Plan- 2016


1) If the election of a person as President is declared void by the Supreme Court, acts done
by him before the date of such declaration of the Supreme Court are not invalidated and
continue to remain in force
2) Impeachment is a quasi-judicial procedure
3) During impeachment, the President cannot defend himself by taking service of the
Attorney General or the Solicitor General or the Additional Solicitors General of India
Which of the statements given above is/are incorrect?
a) 1 and 3 only
b) 1 only
c) 2 only
d) 3 only
Q.37) Solution (d)
Explanation:
If the election of a person as President is declared void by the Supreme Court, acts done
by him before the date of such declaration of the Supreme Court are not invalidated and
continue to remain in force
Impeachment is a quasi-judicial procedure
During impeachment, the President can defend himself by taking service of the Attorney
General of India or any other lawyer he wishes for
Source: Chapter 17 - President, Laxmikanth
Q.38) Consider the following statements:
1) Central Legislation can over-ride the veto power of President in the case of suspensive
veto
2) State Legislature cannot over-ride the veto power of President in the case of suspensive
veto
Which of the statements given above is/are correct?
a) 1 only
b) 2 only
c) 1 and 2 only
d) None of the above
Q.38) Solution (c)
Explanation:
Suspensive Veto:
The President exercises this veto when he returns a bill for reconsideration of the
Parliament. However, if the bill is passed again by the Parliament with or without

www.iasbaba.com

23

Polity- Subject wise compilation of 60 Day Plan- 2016

amendments and again presented to the President, it is obligatory for the President to
give his assent to the bill. This means that the presidential veto is overridden by a repassage of the bill by the same ordinary majority.
When a bill is reserved by the governor for the consideration of the President, the
President may direct the governor to return the bill (if it is not a money bill) for the
reconsideration of the state legislature. If the bill is passed again by the state legislature
with or without amendments and presented again to the President for his assent, the
President is not bound to give his assent to the bill. This means that the state legislature

Source: Chapter 17 - President, Laxmikanth


Q.39) Consider the case of constitutional amendment bill, where the consent of the state
legislatures is required. Which among the following statements is/are true?
1) The Constitution does not prescribe the time frame within which the state legislatures
should ratify or reject an amendment submitted to them
2) The Constitution is silent on the issue whether the states can withdraw their approval
after according the same
Choose the appropriate code:
a) 1 only
b) 2 only
c) 1 and 2 only
d) None of the above
Q.39) Solution (c)
Explanation:
Self-explanatory (both statements are correct)
Source: Chapter 10 - Amendment of the Constitution, Laxmikanth
Q.40) Recently, there was news that Tamil Nadu government decided to remit the life
sentences of all the seven convicts in the Rajiv Gandhi assassination case. Which among
the following statements is/are true?
1) State has to consult the Centre before releasing prisoners prosecuted by the CBI or
under a Central law
2) Governor of a State has the power to suspend, remit or commute the sentence of any
person convicted of any offence against any law relating to a matter to which the
executive power of the State extends
Choose the appropriate code:

www.iasbaba.com

24

Polity- Subject wise compilation of 60 Day Plan- 2016


a)
b)
c)
d)

1 only
2 only
1 and 2 only
None of the above

Q.40) Solution (c)


Explanation:
Tamil Nadu government decided to remit the life sentences of all the seven convicts in
the Rajiv Gandhi assassination case and release them as they had already served over 24
years in prison.
Under Section 435 of the Cr.PC, the State has to consult the Centre before releasing
prisoners prosecuted by the CBI or under a Central law.
In December last year, the Supreme Court had ruled that the State government had no
power to release the Rajiv Gandhi case convicts without the Centres concurrence.
Article 161: Governor of a State has the power to grant pardons, reprieves, respites or
remissions of punishment or to suspend, remit or commute the sentence of any person
convicted of any offence against any law relating to a matter to which the executive
power of the State extends.
Please note that President can grant pardon to a person awarded death sentence. But
Governor of State does not enjoy this power.
Source: Chapter 17 - President, Laxmikanth
TN seeks Centre's views on freeing Rajiv case convicts - The Hindu
Q.41) With reference to Union Government, consider the following statements:
1) The Ministries Departments of the Government of India are created by the Prime
Minister on the advice of the Cabinet Secretary
2) Each of the Ministries is assigned to a Minister by the President of India on the advice of
the Prime Minister
Which of the statements given above is/are correct?
a) 1 only
b) 2 only
c) 1 and 2 only
d) None of the above
Q.41) Solution (b)
Explanation:
The Ministries/Departments of the Government of India are created by the President on
the advice of the Prime Minister under the Government of India (Allocation of Business)
Rules, 1961

www.iasbaba.com

25

Polity- Subject wise compilation of 60 Day Plan- 2016

The business of the Government of India are transacted in the ministries/departments,


secretariats and offices (referred to as "Department") as per the distribution of subjects
specified in these Rules.
Each of the Ministry (ies) will be assigned to a Minister by the President on the advice of
the Prime Minister. Each department will be generally under the charge of a Secretary to
assist the Minister on policy matters and general administration.

Source: Chapter 20 Central Council of Ministers, Laxmikanth


UPSC 2009 Question, Explanation link: http://www.eoi.gov.in/ashgabat/?0775?000#
Q.42) Consider the following statements in regard to constitutional amendment bill:
1) The constitutional amendment bill must be passed in each House by a majority of more
than 50 per cent of the total strength, which includes even those members who are
abstaining
2) In case of joint sitting, for the purpose of deliberation and passage of constitutional
amendment bill, the bill must be passed by a simple majority
Which of the statements given above is/are correct?
a) 1 only
b) 2 only
c) 1 and 2 only
d) None of the above
Q.42) Solution (d)
Explanation:
The constitutional amendment bill must be passed in each House by a special majority,
that is, a majority (that is, more than 50 per cent) of the total membership of the House
and a majority of two-thirds of the members of the House present and voting.
In 1st statement, it is explaining about Absolute Majority (hence statement 1 is wrong)
There is no provision of joint sitting, for the purpose of deliberation and passage of
constitutional amendment bill (so statement 2 is also wrong)
Source: Chapter 10 - Amendment of the Constitution, Laxmikanth
Q.43) For which of the following cases, constitutional amendment has been done?
1. Reduction in voting rights from 21 to 18
2. Introducing any language as official language
3. Substituting Odia for Oriya
4. Restrict the size of council of ministers to 15% of legislative members
Select the appropriate code

www.iasbaba.com

26

Polity- Subject wise compilation of 60 Day Plan- 2016


a)
b)
c)
d)

1 and 3
2, 3 and 4
1, 2 and 4
1, 2, 3 and 4

Q.43) Solution (d)


Reduction in voting rights from 21 to 18- 61st Amendment
Introducing any language as official language- Amendment in schedule 8
Substituting Odia for Oriya- Amendment in Schedule
Restrict the size of council of ministers to 15% of legislative members- 91st Amendment
Q.44) Oath or affirmation which says Preserve, Protect and Defend Constitution of India
is taken by?
1. Governor
2. Prime Minister
3. President
4. Vice- President
5. Chief Minister
Select the correct code
a) 1, 2, 4 and 5
b) 2, 3, 4 and 5
c) 1 and 3
d) 3 and 4
Q.44) Solution (c)
Article 60 {Oath or affirmation by the President}
Every President and every person acting as President or discharging the functions of the
President shall, before entering upon his office, make and subscribe in the presence of the
Chief Justice of India or, in his absence, the senior most Judge of the Supreme Court
available, an oath or affirmation in the following form, that is to say - "I, A.B., do swear in
the name of God / solemnly affirm that I will faithfully execute the office of President (or
discharge the function of the President) of India and will to the best of my ability preserve,
protect and defend the Constitution and the law and that I will devote myself to the
service and well-being of the people of India."
Article 159 {Oath or affirmation by the Governor}
Every Governor and every person discharging the functions of the Governor shall, before
entering upon his office, make and subscribe in the presence of the Chief Justice of the High
court exercising jurisdiction in relation to the State, or, in his absence, the senior most Judge
of that Court available, an oath or affirmation in the following form, that is to say - "I, A.B.,
do

www.iasbaba.com

27

Polity- Subject wise compilation of 60 Day Plan- 2016


Swear in the name of God / solemnly affirm that I will faithfully execute the office of
Governor (or discharge the functions of the Governor) of ...................... (name of the
State) and will to the best of my ability preserve, protect and defend the Constitution and
the law and that I will devote myself to the service and well-being of the people of
....................
(name of the State)."
Q.45) Consider the following regarding Attorney General of India
1. He belongs to the category of Government Servants
2. He is full time counsel of Government of India
3. He doesnt enjoy privileges and immunities that are available to a Member of
Parliament.
Select the incorrect code
a) Only 3
b) 1 and 2
c) 2 and 3
d) 1, 2 and 3
Q.45) Solution (d)
In the performance of his official duties, the Attorney General has the right of audience in all
courts in the territory of India. Further, he has the right to speak and to take part in the
proceedings of both the Houses of Parliament or their joint sitting and any committee of the
Parliament of which he may be named a member, but without a right to vote. He enjoys all
the privileges and immunities that are available to a Member of Parliament.
However, the Attorney General is not a full-time counsel for the Government. He does not
fall in the category of government servants. Further, he is not debarred from private legal
practice.
Source: Chapter on Attorney General of India, Laxmikanth
Q.46) The present relationship between the President and the council of ministers is
governed by the provisions of:
a) 42nd Amendment Act
b) 48th Amendment Act
c) 54th Amendment Act
d) 44th Amendment Act
Q.46) Solution (d)
Q.47) Consider the following statements:

www.iasbaba.com

28

Polity- Subject wise compilation of 60 Day Plan- 2016


1. An amendment to the Constitution of India can be initiated by an introduction of a bill in
the Lok Sabha only.
2. If such an amendment seeks to make changes in the federal character of the Constitution,
the amendment also requires to be ratified by the legislature of all the States of India.
Which of the statements given above is/are correct?
a) 1 only
b) 2 only
c) Both 1 and 2
d) Neither 1 nor 2
Q.47) Solution (d)
Q.48) Consider the following statements in regard to the President of India:
1. President is the highest decision-making authority in our politico-administrative system.
2. President can require the Prime Minister to submit, for consideration of the council of
ministers, any matter on which a decision has been taken by a minister but, which has
not been considered by the council.
3. President can make regulations for the peace, progress and good government of the
Andaman and Nicobar Islands, Lakshadweep, Dadra and Nagar Haveli and Daman and
Diu.
4. In the case of Puducherry and Delhi, the President can legislate by making regulations
but only when the assembly is suspended or dissolved.
Which of the statements given above are correct?
a) 1, 3 and 4
b) 2 and 3
c) 1, 2 and 3
d) All of the above
Q.48) Solution (b)
Explanation:
Cabinet is the highest decision-making authority in our politico-administrative system.
Only in the case of Puducherry (not Delhi), the President can legislate by making
regulations but only when the assembly is suspended or dissolved.
Q.49) Consider the following statements in regard to the removal of Vice President of
India:
1. The Vice-President may be removed on any ground considered sufficient by the two
Houses of the Parliament

www.iasbaba.com

29

Polity- Subject wise compilation of 60 Day Plan- 2016


2. In both the cases of removal of the President and the Vice-President, the power of
removal is lodged in the two Houses of Parliament
3. A Vice President may be removed from his office by a resolution of the council of States
passed by an absolute majority and agreed to by the House of the People
4. A Vice President may be removed from his office by a resolution of the council of States
passed by a special majority and agreed to by the House of the People
Which of the statements given above is/are incorrect?
a) 2 and 4 only
b) 3 and 4 only
c) 3 only
d) 4 only
Q.49) Solution (b)
Explanation:
The Vice-President may be removed on any ground considered sufficient by the two
Houses of the Parliament (as no ground has been mentioned in the Constitution for his
removal).
In both the cases of removal of the President and the Vice-President, the power of
removal is lodged in the two Houses of Parliament, and coordinated action is called for
on their part to effect the removal of either dignitary.
Article 67 (b) states that a Vice President may be removed from his office by a
resolution of the council of States passed by a majority of all the then members of the
council and agreed to by the House of the People
In other words, Vice President can be removed by a resolution of the Rajya Sabha passed
by an effective majority (more than 50% of effective membership(total membership Vacant seats)) and agreed to by a simple majority (50% of present and voting members)
of the Lok Sabha.
Reasons:
1) Special Majority not required:
A major difference in the two procedures (removal of President and removal of Vice
President) lies in the requirement of two-thirds majority of the total membership of each
House for passing the two resolutions which are necessary for the impeachment of a
President. For the removal of a Vice-President from office, a resolution passed by a
majority of all the then members of the Council and agreed to by the House of the People
is sufficient.
Proof:
http://14.139.60.114:8080/jspui/bitstream/123456789/15156/1/015_Procedure%20for%20
the%20removal%20of%20the%20Vice%20President%20of%20India%20(107-111).pdf (Page
no.3 First para, last lines)

www.iasbaba.com

30

Polity- Subject wise compilation of 60 Day Plan- 2016


2) Absolute Majority (no way):
Definition of Absolute Majority - It is the majority of more than 50% of the total strength of
the House (which includes even those members who are abstaining)
For ex, in case of RS - it has total strength of 245 members, therefore 123 and above shall be
an absolute majority
But what Article 67(b) says - passed by a majority of all the then members
For ex, 245 is total RS members, but let us consider there were 15 vacancies, so 230
members shall be the effective strength
So, majority of all the then members - i.e. more than 50% of 230 = 116 or more ---- is called
the effective majority
Therefore, removal of VP requires "effective majority" (not special nor absolute nor simple)
in RS
3) Not simple majority
Reason: Let us consider total members of both the House (245 RS + 545 LS) = 790 members
In this, for ex, we shall consider 700 members are present
Now, out of this 700 members present, let us assume total number of the members of
Parliament present and voting is 500
So simple majority is more than 50% of present and voting. i.e. 251 or above will be the
simple majority
Q.50) Which among the following qualifications for a person to be chosen as a member of
the Parliament is laid down by the Constitution of India?
1. He must be not less than 30 years of age in the case of the Rajya Sabha and not less than
25 years of age in the case of the Lok Sabha.
2. He must be registered as an elector for a parliamentary constituency. This is same in the
case of both, the Rajya Sabha and the Lok Sabha.
3. He must make and subscribe to an oath or affirmation before the person authorised by
the election commission for this purpose, according to the form prescribed in the Fourth
Schedule.
Choose the appropriate code:
a) 1 only
b) 2 only
c) 1 and 2 only
d) All of the above
Q.50) Solution (a)
Explanation:
He must be not less than 30 years of age in the case of the Rajya Sabha and not less than
25 years of age in the case of the Lok Sabha.

www.iasbaba.com

31

Polity- Subject wise compilation of 60 Day Plan- 2016

He must be registered as an elector for a parliamentary constituency. This is same in the


case of both, the Rajya Sabha and the Lok Sabha. (This statement is incorrect because
the Constitution does not lay down this qualification, it is provided in the Representation
of People Act (1951))
He must make and subscribe to an oath or affirmation before the person authorised by
the election commission for this purpose, according to the form prescribed in the Third
Schedule (not fourth).

Q.51) Consider the following statements and select the correct statement/s from the
codes given below:
1. Presiding officer decides on questions as to disqualifications of members of the
Parliament, in consultation with the Election Commission
2. Finance Minister causes to be laid before the Parliament the annual financial statement
3. No demand for a grant can be made except on Presidents recommendation
Choose the appropriate code:
a) 1 and 3 only
b) 2 and 3 only
c) 3 only
d) All of the above
Q.51) Solution (c)
Explanation:
President decides on questions as to disqualifications of members of the Parliament, in
consultation with the Election Commission
President causes to be laid before the Parliament the annual financial statement
No demand for a grant can be made except on Presidents recommendation
Q.52) Which among the following statements is not true in regard to Presidents powers?
1. Presidents power to give or not to give assent to the bills passed by the Parliament is a
discretionary power, except in case of money bills and constitutional amendment bills.
2. The object of conferring this discretionary power on the President is to prevent hasty
and ill-considered legislation by the Parliament and to prevent a legislation which may
be unconstitutional.
3. It is not obligatory for the President to give his assent even if the bill is again passed by
the state legislature and sent again to him for his consideration.
Choose the appropriate code:
a) 1 only
b) 1 and 2 only
c) 2 only

www.iasbaba.com

32

Polity- Subject wise compilation of 60 Day Plan- 2016


d) 1 and 3 only
Q.52) Solution (b)
Explanation:
President has no such discretionary powers (hence, statement 1 and 2 is wrong)
President can act on his discretion (that is, without the advice of the ministers) under
the following situations:
1) Appointment of Prime Minister when no party has a clear majority in the Lok Sabha or
when the Prime Minister in office dies suddenly and there is no obvious successor.
2) Dismissal of the council of ministers when it cannot prove the confidence of the Lok
Sabha.
3) Dissolution of the Lok Sabha if the council of ministers has lost its majority.
With respect to the government bills when the cabinet resigns (after the passage of the
bills but before the assent by the President) and the new cabinet can advise the
President not to give his assent to such bills.

Doubts: In Q5, the power of president to give or not to give assent to the bills passed by
parliament is a discretionary power.
Ex: Pocket Veto and Absolute Veto. No council or person advises him to do that. He decides
that on his own right??
Clarification: Statement (1) Presidents power to give or not to give assent to the bills
passed by the Parliament is a discretionary power, except in case of money bills and
constitutional amendment bills.
The statement does not specifically mention about particular veto (pocket or absolute veto)
Moreover, there is no mention of Presidents power to give or not to give assent to the bills
is his discretionary power.
*If it is mentioned anywhere, please enlighten us sharing the link. We are open to learning.
In addition, Laxmikanth says thisPresident has no constitutional discretion, he has some situational discretion. In other
words, the President can act on his discretion (that is, without the advice of the ministers)
under the following situations:
Appointment of Prime Minister when no party has a clear majority in the Lok Sabha or
when the Prime Minister in office dies suddenly and there is no obvious successor.
Dismissal of the council of ministers when it cannot prove the confidence of the Lok
Sabha.
Dissolution of the Lok Sabha if the council of ministers has lost its majority.

www.iasbaba.com

33

Polity- Subject wise compilation of 60 Day Plan- 2016

Q.53) Whenever a bill seeking to replace an ordinance is introduced in the House


1. A statement explaining the circumstances that had necessitated immediate legislation
by ordinance should also be placed before the House.
2. A statement explaining why the concerned House is seeking to replace an ordinance
should be placed before the President.
3. The ordinance may cease to operate even earlier than the prescribed expiry of six
weeks.
Which of the statements given above is/are correct?
a) 1 only
b) 2 only
c) 3 only
d) Both 2 and 3
Q.53) Solution (a)
Explanation:
Only 1st statement is correct - Self-explanatory
Doubt: Q6-Ordinance can lapse even before prescribed 6 weeks....if president withdraw the
ordinance before the prescribed period...so third statement should be correct
Explanation:
Whenever a bill seeking to replace an ordinance is introduced in the House, a statement
explaining the circumstances that had necessitated immediate legislation by ordinance
should also be placed before the House.
Statement (2) is wrong as there is no such condition
Statement (3) is wrong because - ordinance may cease to operate even earlier than the
prescribed expiry of six weeks when both the house passes a resolution disapproving the
ordinance (and not whenever a bill seeking to replace an ordinance is introduced in the
House)
*Please be aware while reading the question
Statement (3) when read alone, it is absolutely correct. But while read along with the
question, it is wrong.
Q.54) Consider the following differences with regard to the President and the Governor of
India:
1. Governor has both constitutional and situational discretion power
2. President has no constitutional discretion, he has only some situational discretion

www.iasbaba.com

34

Polity- Subject wise compilation of 60 Day Plan- 2016


3. The pardoning power of the President is independent of the judiciary and is an executive
power
4. Both the Governor and the President have concurrent power in respect of suspension,
remission and commutation of death sentence
Which of the statements given above is/are correct?
a) 1, 2 and 4 only
b) 3 only
c) 3 and 4
d) All of the above
Q.54) Solution (d)
Explanation:
All the statements given are correct Self explanatory
Governor has both constitutional and situational discretion power. For instance:
(Examples of constitutional discretion)
Reservation of a bill for the consideration of the President
Recommendation for the imposition of the Presidents Rule in the state
(Examples of situational discretion)
Appointment of chief minister when no party has a clear-cut majority in the state
legislative assembly or when the chief minister in office dies suddenly and there is no
obvious successor
Dismissal of the council of ministers when it cannot prove the confidence of the state
legislative assembly
President has no constitutional discretion, he has some situational discretion. In other
words, the President can act on his discretion (that is, without the advice of the ministers)
under the following situations:
Appointment of Prime Minister when no party has a clear majority in the Lok Sabha or
when the Prime Minister in office dies suddenly and there is no obvious successor.
Dismissal of the council of ministers when it cannot prove the confidence of the Lok
Sabha.
Dissolution of the Lok Sabha if the council of ministers has lost its majority.
Doubts:- it should be judicial powers not executive powers of president
Explanation:
PARDONING POWER OF THE PRESIDENT
Article 72 of the Constitution empowers the President to grant pardons to persons who
have been tried and convicted of any offence in all cases where the:
1. Punishment or sentence is for an offence against a Union Law;
2. Punishment or sentence is by a court martial (military court); and
3. Sentence is a sentence of death.
www.iasbaba.com

35

Polity- Subject wise compilation of 60 Day Plan- 2016

The pardoning power of the President is independent of the Judiciary; it is an executive


power. But, the President while exercising this power, does not sit as a court of appeal. The
object of conferring this power on the President is two-fold: (a) to keep the door open for
correcting any judicial errors in the operation of law; and, (b) to afford relief from a
sentence, which the President regards as unduly harsh.
Q.55) Consider the following statements in regard to the Prime Minister of India:
1. The Prime Minister holds office for a term of five years from the date on which he enters
upon his office.
2. However, if he loses the confidence of the Lok Sabha, he must resign or the President
can dismiss him.
3. Prime Minister gets the salary and allowances that are payable to a member of
Parliament, only his sumptuary allowance varies
Which of the statements given above is/are correct?
a) 1 and 2 only
b) 2 only
c) 2 and 3 only
d) All of the above
Q.55) Solution (c)
Explanation:
The term of the Prime Minister is not fixed and he holds office during the pleasure of the
President.
Q.56) Consider the following statements:
1. In India the President is only a nominal executive and the real powers are vested in the
prime minister.
2. The executive power of the Union shall be vested in Prime Minister and shall be
exercised by him either directly or through officers subordinates to him.
3. Council of ministers headed by the Prime Minister advices the President with regard to
summoning and proroguing of the sessions of the Parliament.
4. Orders and other instruments made and executed in the name of the President shall be
authenticated in such manner as may be specified in rules made by the Parliament.
Which of the statements given above is/are incorrect?
a) 2 only
b) 1 and 2 only
c) 2 and 4 only
d) All of the above

www.iasbaba.com

36

Polity- Subject wise compilation of 60 Day Plan- 2016

Q.56) Solution (d)


Explanation:
President is only a nominal executive and the real powers are vested in the council of
ministers headed by the prime minister.
The executive power of the Union shall be vested in President and shall be exercised by
him either directly or through officers subordinates to him in accordance with this
Constitution (Article 53).
Prime Minister advises the President with regard to summoning and proroguing of the
sessions of the Parliament.
Orders and other instruments made and executed in the name of the President shall be
authenticated in such manner as may be specified in rules to be made by the President.
Doubts and clarification: Whatever your doubts are/were, the above question, answer and
explanation to it are absolutely correct.
*Please learn to use CoM headed by Prime Minister wherever it is mentioned as such. Dont
assume it as Prime Minister. Same goes with CoM and Cabinet. Dont interchange and use it.
Few questions similar on above lines have been asked by UPSC. For instance, 2015 UPSC CSE
Prelims had a question:
Consider the following statements:
1) The Executive Power of the Union of India is vested in the Prime Minister.
2) The Prime Minister is the ex officio Chairman of the Civil Services Board.
Which of the statements given above is/are correct?
a) 1 only
b) 2 only
c) Both 1 and 2
d) Neither 1 nor 2
Solution (d)
Explanation:
The Executive Power of the Union of India is vested in the President.
The Cabinet Secretary (and not the Prime Minister) is the ex-officio head of the Civil
Services Board.

Q.57) Consider the following in regard to President of India:


1. A person to be eligible for election as President should have completed 35 years of age,
i.e, should be qualified for election as a member of the second chamber.
2. The salary and allowances of the president are charged from Consolidated Fund of India.

www.iasbaba.com

37

Polity- Subject wise compilation of 60 Day Plan- 2016


3. President may be elected for any number of terms
Which of the statements given above is/are correct?
a) 1 and 2 only
b) 2 and 3 only
c) 1 and 3 only
d) All of the above
Q.57) Solution (b)
A person to be eligible for election as President should have completed 35 years of age,
i.e, should be qualified for election as a member of the Lok Sabha (not Rajya Sabha).
Q.58) Which among the following is/are correct statements in regard to Council of
ministers and Cabinet?
1. Cabinet functions are determined by the Council of ministers
2. Council of ministers supervises the implementation of its decisions by the cabinet
Choose the appropriate code:
a) 1 only
b) 2 only
c) Both 1 and 2
d) Neither 1 nor 2
Q.58) Solution (d)
Council of ministers functions are determined by the cabinet
Council of ministers implements the decisions taken by the cabinet. Cabinet supervises
the implementation of its decisions by the council of ministers
Q.59) Consider the following statements in regard to Council of Ministers:
1. When the Lok Sabha passes a no-confidence motion against the council of ministers, all
the ministers have to resign including those ministers who are from the Rajya Sabha.
2. The council of ministers can advise the president to dissolve the Lok Sabha on the
ground that the House does not represent the views of the electorate faithfully and call
for fresh elections.
3. It is the duty of every minister to stand by cabinet decisions and support them both
within and outside the Parliament. If any minister disagrees with a cabinet decision and
is not prepared to defend it, he must resign.
4. President can remove a minister even at a time when the council of ministers enjoys the
confidence of the Lok Sabha.
Which of the statements given above is/are correct?

www.iasbaba.com

38

Polity- Subject wise compilation of 60 Day Plan- 2016


a)
b)
c)
d)

3 and 4 only
2, 3 and 4 only
1, 2 and 3 only
All of the above

Q.59) Solution (d)


Explanation:
All the given statements are correct Self-explanatory
Q.60) Who among the following hold office during the pleasure of the President?
1) Attorney General
2) Prime Minister
3) Governor
4) Council of Ministers
Choose the appropriate code:
a) 1 only
b) 1, 2 and 3 only
c) 1 and 3 only
d) All of the above
Q.60) Solution (d)
Q.61) Which provision is regarded as the foundation of the Parliamentary system of
government?
a) Interdependence between Legislature and Executive organs
b) The council of ministers shall be collectively responsible to the Lok Sabha
c) The executive power of the Union shall be vested in President and shall be exercised by
him either directly or through officers subordinate to him
d) Lok Sabha is elected directly by the people
Q.61) Solution (b)
Q.62) Consider the below statements in regard to the President of India:
1. President is the supreme executive authority of the Central government and chief
coordinator of Central administration.
2. He is the supreme commander of the defence forces of India.
3. In that capacity, he appoints the chiefs of the Army, the Navy and the Air Force and can
declare war or conclude peace, subject to the approval of the Parliament.
Which of the statements given above is/are correct?

www.iasbaba.com

39

Polity- Subject wise compilation of 60 Day Plan- 2016


a)
b)
c)
d)

1 and 3 only
2 and 3 only
1 and 2 only
None

Q.62) Solution (b)


Explanation:
Statement 1 is not correct - Cabinet is the supreme executive authority of the Central
government and chief coordinator of Central administration.
Q.63) Consider the following:
Term
Meaning
1. Pardon
Completely relief from any punishment
2. Reprieve
Temporary suspension of any punishment
3. Respite
Awarding less sentence
4. Commutation
Changing one punishment to another
Which of the above terms are correctly matched?
a) 1 and 3
b) 2 and 4
c) 1, 2 and 4
d) All of the above
Q.63) Solution (d)
A President is empowered with the power to pardon under Article 72 of the Indian
Constitution. Article 72 says that the President shall have the power to grant pardons,
reprieves, respites or remissions of punishment or to suspend, remit or commute the
sentence of any person convicted of any offence. The meaning of these terms is as follows:
Pardon: Complete pardon
Reprieve: Temporary suspension of sentence
Respite: awarding less sentence
Remission: Reducing amount of sentence
Commutation: Changing one punishment to another.
Q.64) Consider the following statements regarding impeachment of the President of India:
1. The President of India can be impeached from the office for violation of Constitution,
insolvency and using public office for private gains.
2. A motion of impeachment can be initiated only in Loksabha.
3. The proposal to prefer such charge is contained in a resolution which has been moved
after at least fourteen days' notice in writing, signed by not less than one-third of the

www.iasbaba.com

40

Polity- Subject wise compilation of 60 Day Plan- 2016


total number of members of the House, has been given of their intention to move the
resolution.
4. If the resolution is passed by a special majority (two thirds of present and voting), the
President stands impeached from his office.
Which of the above statements is/are incorrect regarding the impeachment of the
President of India?
a) 1 and 2
b) 4 only
c) None of the above
d) All of the above
Q.64) Solution (d)
All of the above statements are incorrect regarding the impeachment of the President of
India.
Procedure for impeachment of the President
(1) When a President is to be impeached for violation of the Constitution, the charge shall
be preferred by either House of Parliament.
(2) No such charge shall be preferred unless(a) The proposal to prefer such charge is contained in a resolution which has been moved
after at least fourteen days' notice in writing, signed by not less than one-fourthof the
total number of members of the House, has been given of their intention to move the
resolution, and
(b) Such resolution has been passed by a majority of not less than two-thirds of the total
membership of the House.
a) (3) When a charge has been so preferred by either House of Parliament, the other House
shall investigate the charge or cause the charge to be investigated and the President
shall have the right to appear and to be represented at such investigation.
b) (4) If as a result of the investigation a resolution is passed by a majority of not less than
two-thirds of the total membership of the House by which the charge was investigated
or caused to be investigated, declaring that the charge preferred against the President
has been sustained, such resolution shall have the effect of removing the President from
his office as from the date on which the resolution is so passed.
Q.65) The Ordinance mechanism has been devised to enable the executive to deal with a
situation that may suddenly and immediately arise when the Parliament is not in session
(either one or both houses). Which of the following statements are correct regarding
Ordinance making in India?
1. Article 123 of Indian Constitution empowers the president to promulgate Ordinances.
2. Maximum life of an ordinance can be 6 months and 6 weeks.

www.iasbaba.com

41

Polity- Subject wise compilation of 60 Day Plan- 2016


3. An Ordinance can only be promulgated on the subjects on which Parliament is
competent to make laws.
Select the code from the following:
a) 1 and 2
b) 2 and 3
c) 1 and 3
d) All of the above
Q.65) Solution (d)
Article 123 of the Indian constitution empowers the President to promulgate ordinances.
An Ordinance promulgated under this article shall have the same force and effect as an Act
of Parliament, but every such Ordinance
Shall be laid before both Houses of Parliament and shall cease to operate at the expiration
of six weeks from the reassembly of Parliament, or, if before the expiration of that period
resolution disapproving it are passed by both Houses, upon the passing of the second of
those resolutions. As maximum gap between two session of a house can be 6 months
therefore maximum life of a ordinance can be 6 months and 6 weeks; and May be
withdrawn at any time by the President.
Q.66) Article 74(1) requires the President to have a Council of Ministers with the Prime
Minister at the head to aid and advice him in the exercise of his power. Which of the
following statements regarding Council of Ministers are correct?
1. Ministers are appointed by the Prime Minister.
2. India has a three-tier ministry consisting of cabinet ministers, ministers of state and the
deputy ministers.
3. The term Cabinet is not present in the original Constitution.
Select the code from the following:
a) 1 and 2
b) 2 and 3
c) 1 and 3
d) All of the above
Q.66) Solution (b)
Ministers are appointed by the President on the advice of the Prime Minister.
Q.67) Consider the following statements in regard to the conditions of Governors office:
1. Governor is entitled to such emoluments, allowances and privileges as may be
determined by Parliament.

www.iasbaba.com

42

Polity- Subject wise compilation of 60 Day Plan- 2016


2. However, when the same person is appointed as the governor of two or more states,
the emoluments and allowances are determined by the president.
Which of the statements given above is/are correct?
a) 1 only
b) 2 only
c) Both
d) None
Q.67) Solution (a)
Explanation:
Governor is entitled to such emoluments, allowances and privileges as may be
determined by Parliament.
However, when the same person is appointed as the governor of two or more states,
the emoluments and allowances payable to him are shared by the states in such
proportion as determined by the president.
Doubts: BABA, i don't think that second statement of first question is wrong because
ultimately it is president who determines the question of allowances and all. you have
nowhere mentioned that allocation to governor goes either from those two states or not.
Clarification: President only determines the proportion how states should share the
emoluments and allowances. But still the emoluments, allowances and privileges are
determined by the Parliament itself.
Q.68) Consider the following statements in regard to the appointment and oath of the
Governor:
1. Governor should be an outsider, that is, he should not belong to the state where he is
appointed.
2. While appointing the governor, the president is required to consult the chief minister of
the state concerned and the latters advice is binding.
3. The oath of office to the governor is administered by the President or some person
appointed in that behalf by him.
Which of the statements given above is/are correct?
a) 1 only
b) 1 and 2 only
c) 1 and 3 only
d) 3 only
Q.68) Solution (a)

www.iasbaba.com

43

Polity- Subject wise compilation of 60 Day Plan- 2016


Explanation:
While appointing the governor, the president is required to consult the chief minister of
the state concerned and the latters advice is not binding.
The oath of office to the governor is administered by the chief justice of the concerned
state High Court and in his absence, the senior-most judge of that court available.
Doubts: "Governor should be an outsider, that is, he should not belong to the state where
he is appointed." In the above statement,Is there a written rule which disallows a person to
be the governor of a state where he belongs.I think the above statement is just a convention
followed. Please clarify IASbaba
Clarification: Yes, many of your arguments are correct. The 1st statement is just a
convention and is not written explicitly anywhere as such.
The general understanding is that he should not belong to the state where he is
appointed.
But when you consider the options given, the most appropriate answer will be option (a) 1
only.
Q. 69) Consider the following statements:
1. State council of ministers advises the governor with regard to the appointment of
important officials like advocate general, chairman and members of the state public
service commission, state election commissioner, and so on.
2. Presiding officer announces the government policies on the floor of the house.
Which of the statements given above is/are incorrect?
a) 1 only
b) 2 only
c) Both
d) None
Q.69) Solution (c)
Explanation:
Chief Minister advises the governor with regard to the appointment of important
officials like advocate general, chairman and members of the state public service
commission, state election commissioner, and so on.
Chief Minister announces the government policies on the floor of the house.
Doubts: Dear IASbaba
when nothing is mentioned in the constitution regarding the advice for appointment, it shall
always be considered as given by "Council of Minister". Since Article 165 merely says
appointment by Governor of advocate general, it will be as "appointment by governor, on

www.iasbaba.com

44

Polity- Subject wise compilation of 60 Day Plan- 2016


aid and advice of CoM, not by Prime minister". Even DD basu says this which is higher
authority than Laxmikant.
There were many questions on this point in your tests, So it need to be clarified as it cannot
be differentiated between advice given by CM and CoM, after all CM is also part of CoM.
Clarification: Chapter 27 Chief Minister in Laxmikanth (Chapter 25 in old book) clearly
mentions under the heading In Relation to the Governor that the Chief Minister advises
the governor with regard to the appointment of important officials like advocate general,
chairman and members of the state public service commission, state election commissioner,
and so on.
Q.70) Which among the following given statements is/are the constitutional discretion
that Governor exercise?
1. Determining the amount payable by the Government of Assam, Meghalaya, Tripura and
Mizoram to an autonomous Tribal District Council as royalty accruing from licenses for
mineral exploration
2. Seeking information from the chief minister with regard to the administrative and
legislative matters of the state
3. The governor has constitutional discretion while exercising his functions as the
administrator of an adjoining union territory
Choose the appropriate code:
a) 1 and 2 only
b) 3 only
c) 1 and 3 only
d) All of the above
Q.70) Solution (d)
Explanation:
All the given statements are correct Self explanatory
If any dispute arises as to the share of such royalties to be made over to a district
council, it shall be referred to the governor for determination and the amount
determined by the governor in his discretion shall be deemed to be the amount payable
to the district council and the decision of the governor shall be final
Q.71) Consider the following statements in regard to the Chief Minister of a State:
1. CM is the chief spokesman of the state government
2. He acts as a chairman of the concerned zonal council by rotation, holding office for a
period of one year at a time
3. He is the member of the Inter-State Council and the National Development Council, both
headed by the prime minister

www.iasbaba.com

45

Polity- Subject wise compilation of 60 Day Plan- 2016

Which of the statements given above is/are correct?


a) 1 and 2 only
b) 1 and 3 only
c) 2 and 3 only
d) All of the above
Q.71) Solution (b)
Explanation:
Union home minister is the chairman of all the zonal councils.
CM acts as a vice-chairman of the concerned zonal council by rotation, holding office for
a period of one year at a time
Q.72) Which among the below given statements is/are correct in regard to Governor?
1. The term of the Governor is not fixed by the Constitution and he holds office during the
pleasure of the President
2. Also the Constitution does not lay down any grounds upon which a governor may be
removed by the President
Select the correct code:
a) 1 only
b) 2 only
c) Both
d) None
Q.72) Solution (b)
Explanation:
A governor holds office for a term of five years from the date on which he enters upon
his office. However, this term of five years is subject to the pleasure of the President.
Further, he can resign at any time by addressing a resignation letter to the President.
Q.73) Bicameral legislatures have been provided in some states under the Constitution. In
case of a deadlock between the two Houses in such States
a) Joint sitting is called by the Governor and the decision by the majority is taken as final
decision
b) Decision of the Governor is final
c) The opinion of the Legislative Assembly is taken as final after a lapse of specified period
d) The matter is referred to the President for decision
Q.73) Solution (c)
Explanation:

www.iasbaba.com

46

Polity- Subject wise compilation of 60 Day Plan- 2016

Self explanatory

Q.74) Which among the following are the powers of the Governor?
1. He appoints the vice-chancellors of universities in the state
2. He appoints the chief minister and other ministers including Tribal Welfare minister in
the states of Chattisgarh, Jharkhand, Madhya Pradesh and Odisha
3. He appoints the state election commissioner and determines his conditions of service
and tenure of office
4. He decides on the question of removal of members of the state election commission in
consultation with the Election Commission
Choose the appropriate code:
a) 1 and 2 only
b) 1 and 4 only
c) 1, 2 and 3 only
d) All of the above
Q.74) Solution (c)
Explanation:
He appoints the state election commissioner and determines his conditions of service
and tenure of office. However, the state election commissioner can be removed only in
like manner and on the like grounds as a judge of a high court.
Q.75) Consider the following statements in regard to the strength of the state council of
ministers:
1. The total number of ministers, including the chief minister, in the council of ministers in
a state shall not exceed 15 per cent of the total strength of the legislative assembly of
that state.
2. But, the number of ministers, including the chief minister, in a state shall not be less
than 12.
Which of the statements given above is/are not correct?
a) 1 only
b) 2 only
c) Both
d) None
Q.75) Solution (d)
Explanation:
Both the given statements are correct

www.iasbaba.com

47

Polity- Subject wise compilation of 60 Day Plan- 2016


Q.76) Governor has certain special responsibilities to discharge according to the directions
issued by the President. In which among the following states he can be directed to
discharge special responsibilities?
1. Karnataka
2. Telangana
3. Gujarat
4. Sikkim
5. Assam
Choose the appropriate code:
a) 1, 3, 4 and 5 only
b) 4 and 5 only
c) 1, 3 and 4 only
d) 1, 2 and 3 only
Q.76) Solution (a)
Explanation:
Governor has certain special responsibilities to discharge according to the directions
issued by the President. They are given below:
1. MaharashtraEstablishment of separate development boards for Vidarbha and
Marathwada.
2. GujaratEstablishment of separate development boards for Saurashtra and Kutch.
3. NagalandWith respect to law and order in the state for so long as the internal
disturbance in the Naga HillsTuensang Area continues.
4. AssamWith respect to the administration of tribal areas.
5. ManipurRegarding the administration of the hill areas in the state.
6. SikkimFor peace and for ensuring social and economic advancement of the different
sections of the population.
7. Arunachal PradeshWith respect to law and order in the state.
8. Karnataka Establishment of a separate development board for Hyderabad-Karnataka
region
Doubts: babajiii please solve this telangana issue too.. whtr governor has the special
responsibility towards it???
Clarification: The Centre has cited provisions of the AP Reorganisation Act, 2014, which
grants special powers to the governor to ensure security in Hyderabad, the shared capital
for 10 years.
Under Section 8, in common capital area, the Governor shall have special responsibility for
the security of life, liberty and property of all those who reside in the city and in discharge of

www.iasbaba.com

48

Polity- Subject wise compilation of 60 Day Plan- 2016


the functions, the Governor shall, after consulting the Council of Ministers of the state of
Telangana, exercise his individual judgement as to the action to be taken.
The Act says the governors decision is final and the validity of anything he or she does
cannot be called into question.
Link:
http://www.deccanchronicle.com/150616/nation-current-affairs/article/securitypeople-hyderabad-governors-responsibility-ap-minister
Q.77) Consider the following statements in regard to the Governor:
1. He is the constitutional head of the state as well as the representative of the Centre
2. He is the chairman of the State Planning Board
3. He acts as the chancellor of universities in the state
4. He is the political head of the services
Which of the statements given above is/are correct?
a) All of the above
b) 1, 3 and 4 only
c) 1 and 3 only
d) 1 only
Q.77) Solution (c)
Explanation:
Chief Minister is the chairman of the State Planning Board and also the political head of
the services
Q.78) Which among the below given statements is/are correct with regard to legislative
councils?
1. The Constitution provides for the abolition or creation of legislative councils in states
2. Accordingly, the Parliament can abolish a legislative council or create it, if the legislative
assembly of the concerned state passes a resolution to that effect
3. Such a specific resolution must be passed by the state assembly by a special majority
4. This Act of Parliament is not to be deemed as an amendment of the Constitution for the
purposes of Article 368 and is passed like an ordinary piece of legislation (i.e, by simple
majority)
Select the appropriate code:
a) 1, 3 and 4
b) 2 and 3
c) 1, 2 and 3
d) All of the above
Q.12)78olution (d)

www.iasbaba.com

49

Polity- Subject wise compilation of 60 Day Plan- 2016


Explanation:
All the given statements are correct and self-explantory
Q.79) Which among the following statements is/are true in regard to Governor?
1. Governor is consulted by the president while appointing the judges of the concerned
state high court.
2. Governor appoints persons to the judicial service of the state (including district judges)
in consultation with the state high court and the State Public Service Commission.
Select the correct code:
a) 1 only
b) 2 only
c) Both
d) None
Q.79) Solution (a)
Explanation:
Governor appoints persons to the judicial service of the state (other than district judges)
in consultation with the state high court and the State Public Service Commission.
Doubts: answer should be (c) Please correct me if i understand it wrongly.
Clarification: Clause (1) of Article 233 says that appointments of persons to be, and the
posting and promotion of, district judges in any State shall be made by the Governor of the
State in consultation with the High Court exercising jurisdiction in relation to such State.
Article 234 says that appointments of persons other than district judges to the judicial
service of a State shall be made by the Governor of the State in accordance with the rules
made by him in that behalf after consultation with the State Public Service Commission and
with the High Court exercising jurisdiction in relation to such State.
Q.80) Which among the below given statements is/are correct in regard to state council of
ministers and central council of ministers?
1. Unlike the Centre, there is no provision in the Constitution for the system of legal
responsibility of the minister in the states.
2. Like at the Centre, in the states too, the council of ministers consists of three categories
of ministers, namely, cabinet ministers, ministers of state, and deputy ministers.
3. The difference between them lies in their respective ranks, emoluments, and political
importance.
Choose the appropriate code:
a) 1 and 2 only

www.iasbaba.com

50

Polity- Subject wise compilation of 60 Day Plan- 2016


b) 2 and 3 only
c) 1 and 3 only
d) All of the above
Q.80) Solution (b)
Explanation:
As at the Centre, there is no provision in the Constitution for the system of legal
responsibility of the minister in the states.
Q.81) Consider the following statements in regard to Deputy Ministers:
1. Deputy Ministers can either be given independent charge of departments or can be
attached to cabinet ministers.
2. However, they are not members of the cabinet and do not attend the cabinet meetings
unless specially invited when something related to their departments are considered by
the cabinet.
Which of the statements given above is/are correct?
a) 1 only
b) 2 only
c) Both
d) None
Q.81) Solution (d)
Explanation:
Deputy Ministers are not given independent charge of departments. They are attached
to the cabinet ministers and assist them in their administrative, political and
parliamentary duties. They are not members of the cabinet and do not attend cabinet
meetings.
Q.82) In which of the following Union Territories, the administrator of the UT is called a Lt.
Governor?
1. Andaman and Nicobar Islands
2. Daman and Diu
3. Delhi
4. Pondicherry
5. Lakshadweep Islands
Select the code from the following:
a) 3 and 4
b) 1,3 and 4
c) 3,4 and 5

www.iasbaba.com

51

Polity- Subject wise compilation of 60 Day Plan- 2016


d) All of the above
Q.82) Solution (b)
The Union Territories are administered by the President through an administrator, who is
appointed by him with a suitable designation. This designation is called either Lieutenant
Governor or Chief Commissioner or Administrator. In Andaman & Nicobar Islands,
Puducherry and Delhi, administrator is called Lt. Governor, while in Chandigarh, Dadra &
Nagar Haveli, Daman & Diu and Lakshadweep he/ she is known as Administrator. The
President may appoint a Governor of an adjoining state as administrator of a Union
territory. In such case the Governor works independently with regard to the administration
of the Union Territory.
Q.83) Consider the following statements regarding the Writ Jurisdiction of High courts:
1. Writs come under the Original jurisdiction of High Court.
2. High Court has been empowered to issue writs for the enforcement of Fundamental
Rights and for other purposes.
Which of the above statements are correct?
a) 1 only
b) 2 only
c) Both 1 and 2
d) Neither 1 nor 2
Q.83) Solution (c)
Writs come under the original jurisdiction of Supreme court as well as the High Court. There
is something called an original jurisdiction and Original and exclusive jurisdiction. This is
original but not exclusive.
The High Court has been empowered to issue writs of habeas corpus, mandamus, and
prohibition certiorari and quo warranto for the enforcement of the fundamental rights and
for other purposes. The Supreme Court can issue the writs only for the enforcement of
fundamental rights and not for other purposes. The power of the High Court to issue writs in
the nature of habeas corpus cannot be curtailed even during emergency.
Q.84) Consider the following statements:
1. It is constitutionally mandatory for the President to consult the Chief Minister of a
respective state before appointing a Governor.
2. The Governor can be removed before his term by the Legislative Assembly of a state by
the process of impeachment.
3. A governor may be simultaneously assigned more than one states.
Which of the above statements are incorrect?

www.iasbaba.com

52

Polity- Subject wise compilation of 60 Day Plan- 2016


a)
b)
c)
d)

1 and 2
2 and 3
1 and 3
All of the above

Q.84) Solution (a)


A Governor is appointed on the advice of the Union Council of Ministers, or in reality on the
advice of the Prime Minister. For the President to consult the Chief Minister of the
concerned state, before the appointment of a Governor is not a constitutional requirement.
But a healthy convention grew up that the Chief Minister was consulted. But in the case of
appointment of several Governors, this convention has not been obeyed.
A Governor may be simultaneously assignee to more than one state. Thus, on many
occasions, the Governor of Assam was also the Governor of Meghalaya, Nagaland, Tripura,
etc.
A Governor is appointed for a period of 5 years. The President, if he so pleases may extend
his tenure. A vacancy in the Governors post arises, if:
(a) when the Governor completes his tenure,
(b) resigns,
(c) dies in office, and
(d) is removed from office by the President.
The Governors also have often been transferred from one state to another.
Q.85) Consider the following statements:
1. According to the preamble of the Constitution of Kashmir, Kashmir is a temporary part
of India.
2. In case of J and K, the residuary powers belong to the states legislature.
3. According to Indian Constitution, article 370 is a temporary provision granting special
autonomous status to Jammu and Kashmir.
Which of the above statements are correct?
a) 1 and 2
b) 2 and 3
c) 1 and 3
d) All of the above
Q.85) Solution (b)
According to the constitution of Kashmir, Kashmir is an integral part of India.
Q.86) Consider the following statements:

www.iasbaba.com

53

Polity- Subject wise compilation of 60 Day Plan- 2016


1. State Election Commission is responsible for conducting the elections for Legislative
Assemblies.
2. Election Commission appoints tribunals for the decision of doubts and disputes arising
out of or in connection with election to parliament and State Legislatures.
3. Only two Union Territories have Legislative Assemblies.
Which of the above statements are correct?
a) 1 and 2
b) 2 and 3
c) 1 and 3
d) All of the above
Q.86) Solution (b)
The main duties of the Election Commission are:
(1) To superintend, direct, control and conduct all elections to Parliament and State
Legislatures as also to the office of the President and Vice- President of India.
(2) to lay down general rules for election.
(3) to determine constituencies and to prepare electoral rolls.
(4) to give recognition to political parties.
(5) to allot election symbols to different political parties and individual contestants.
(6) to appoint tribunals for the decision of doubts and disputes arising out of or in
connection with election to parliament and State Legislatures.
Delhi and Pondicherry are the only two Union Territories to have Legislative Assemblies.
Q.87) The Centre is empowered to give directions to the states with regard to the exercise
of their executive power in which of the following matters?
1) Communication
2) Railways
3) Linguistic minority groups
4) Welfare of the Scheduled Tribes
Select the correct answer using the code given below:
a) 2, 3 and 4
b) 1, 2 and 3
c) 1 and 2
d) All of the above
Q.87) Solution (d)
Explanation:
The Centre is empowered to give directions to the states with regard to the exercise of their
executive power in the following matters:

www.iasbaba.com

54

Polity- Subject wise compilation of 60 Day Plan- 2016


1. the construction and maintenance of means of communication (declared to be of
national or military importance) by the state;
2. the measures to be taken for the protection of the railways within the state;
3. the provision of adequate facilities for instruction in the mother-tongue at the primary
stage of education to children belonging to linguistic minority groups in the state; and
4. the drawing up and execution of the specified schemes for the welfare of the Scheduled
Tribes in the state.
Q.88) The Constitution of India makes which of the following provisions with regard to
interstate comity?
1) Adjudication of inter-state water disputes
2) Establishment of zonal councils
3) Mutual recognition of public acts, records and judicial proceedings
4) Freedom of inter-state trade, commerce and intercourse
Select the correct answer using the code given below:
a) 1 and 2 only
b) 1, 3 and 4 only
c) 1, 2 and 3 only
d) All of the above
Q.88) Solution (b)
Explanation:
The Constitution makes the following provisions with regard to inter-state comity:
1) Adjudication of inter-state water disputes.
2) Coordination through inter-state councils.
3) Mutual recognition of public acts, records and judicial proceedings.
4) Freedom of inter-state trade, commerce and intercourse.
In addition, the zonal councils have been established by the Parliament to promote
interstate cooperation and coordination.
Zonal Councils are not Constitutional bodies akin to the Interstate Council. However,
since they have been established via the part III of the States Reorganization Act of 1956,
they are statutory bodies.
Q.89) Which of the following trends in the working of Indian political system reflects its
federal spirit?
a) Disputes between states over sharing of river water
b) The emergence of regional parties and their coming to power in states
c) Both (a) and (b)
d) None of the above

www.iasbaba.com

55

Polity- Subject wise compilation of 60 Day Plan- 2016


Q.89) Solution (c)
Explanation:
The following trends in the working of Indian political system reflects its federal spirit:
1. Territorial disputes between states, for example, between Maharashtra and Karnataka
over Belgaum;
2. Disputes between states over sharing of river water, for example, between Karnataka
and Tamil Nadu over Cauvery Water;
3. The emergence of regional parties and their coming to power in states like Andhra
4. Pradesh, Tamil Nadu, etc.;
5. The creation of new states to fulfil the regional aspirations, for example, Mizoram or
recently Jharkhand;
6. Demand of the states for more financial grants from the Centre to meet their
developmental needs;
7. Assertion of autonomy by the states and their resistance to the interference from the
Centre;
8. Supreme Courts imposition of several procedural limitations on the use of Article 356
(Presidents Rule in the States) by the Centre.
Q.90) Consider the following statements:
1) The Indian federal system is based on the Canadian model and not on the American
model
2) The Indian federation resembles the Candian federation in its formation, its preference
to the term Union and its centralising tendency
Which of the statements given above is/are correct?
a) 1 only
b) 2 only
c) Both 1 and 2
d) Neither 1 nor 2
Q.90) Solution (c)
Explanation:
The Indian federal system is based on the Canadian model and not on the American
model. The Canadian model differs fundamentally from the American model in so far
as it establishes a very strong centre.
The Indian federation resembles the Candian federation
a) in its formation (i.e., by way of disintegration);
b) in its preference to the term Union (the Canadian federation is also called a Union);
and
c) in its centralising tendency (i.e., vesting more powers in the centre vis-a-vis the states)

www.iasbaba.com

56

Polity- Subject wise compilation of 60 Day Plan- 2016


Q.91) Which of the following statements is/are correct with regard to the provisions of
scheduled and tribal areas of India?
1) The Governor is empowered to direct that an act of Parliament does not apply to a
scheduled area, i.e, states belonging to Schedule V
2) The President may likewise direct that an act of Parliament does not apply to a tribal
area (autonomous district), i.e, states belonging to Schedule VI
Select the correct answer using the code given below:
a) 1 only
b) 2 only
c) Both 1 and 2
d) Neither 1 nor 2
Q.91) Solution (a)
Explanation:
The governor is empowered to direct that an act of Parliament does not apply to a
scheduled area in the state or apply with specified modifications and exceptions.
The Governor of Assam may likewise direct that an act of Parliament does not apply to a
tribal area (autonomous district) in the state or apply with specified modifications and
exceptions. The President enjoys the same power with respect to tribal areas
(autonomous districts) in Meghalaya, Tripura and Mizoram.
Q.92) Consider the following statements:
1) In case of a conflict between the Central law and the state law on a subject enumerated
in the Concurrent List, the Central law prevails over the state law
2) If the state law has been reserved for the consideration of the president and has
received his assent, then the state law prevails in that state
3) It means, only in the above case [statement (2)], Parliament cannot override over the
state law
Which of the statements given above is/are correct?
a) 1 only
b) 1 and 2 only
c) All of the above
d) None of the above
Q.92) Solution (b)
Explanation:
In case of a conflict between the Central law and the state law on a subject enumerated
in the Concurrent List, the Central law prevails over the state law. But, there is an
exception. If the state law has been reserved for the consideration of the president and
has received his assent, then the state law prevails in that state. But, it would still be

www.iasbaba.com

57

Polity- Subject wise compilation of 60 Day Plan- 2016


competent for the Parliament to override such a law by subsequently making a law on
the same matter.
Q.93) The Indian Constitution is unitary in spirit because
a) Parliament can make laws for the states under some circumstances
b) All India Services are provided for
c) Only the Union Parliament has the power to propose amendment in the Constitution
d) All of the above are true
Q.93) Solution (d)
Explanation:
All the given statements are correct self explanatory
Q.94) Which of the following statements is/are correct with regard to the Federal System
of India?
1) Constitution of India provides for a federal system of government in the country
2) Constitution of India established a dual polity consisting the Union at the Centre and
the states at the periphery
Select the correct answer using the code given below:
a) 1 only
b) 2 only
c) Both 1 and 2
d) Neither 1 nor 2
Q.94) Solution (c)
Explanation:
The Constitution of India provides for a federal system of government in the country.
The framers adopted the federal system due to two main reasonsthe large size of the
country and its socio-cultural diversity. They realised that the federal system not only
ensures the efficient governance of the country but also reconciles national unity with
regional autonomy.
However, the term federation has no where been used in the Constitution.
The Constitution established a dual polity consisting the Union at the Centre and the
states at the periphery. Each is endowed with sovereign powers to be exercised in the
field assigned to them respectively by the Constitution.
Doubts: Babaji in Q8 Constitution of India establishes a dual polity alright,but as 73rd CAA
has been implemented doesn't it create a 3 tier structure.I mean Constitution amendment
means Constitution NOW establishes a triple polity??
Clarification: Statement (2) in question has been slightly modified. Instead of establishes,
we have changed it to established

www.iasbaba.com

58

Polity- Subject wise compilation of 60 Day Plan- 2016

Q.95) Consider the following statements:


1) Any legislation passed by the State Legislature for acquisition of private property for
public purposes will not become a law unless it has the assent of the President
2) The legislature of a state can impose reasonable restrictions on the freedom of trade,
commerce and intercourse with that state or within that state in public interest. But, a
bill for this purpose can be introduced in the legislature only with the previous sanction
of the president.
Which of the statements given above is/are correct?
a) 1 only
b) 2 only
c) Both 1 and 2
d) Neither 1 nor 2
Q.95) Solution (c)
Explanation:
Both the given statements are correct self explanatory
Q.96) The objectives (or the functions) of the zonal councils does not include which of the
following:
a) To help in arresting the growth of acute state-consciousness, regionalism, linguism and
particularistic trends
b) To help in removing the after-effects of separation in some cases so that the process of
reorganisation, integration and economic advancement may synchronise
c) To secure some kind of political equilibrium between different regions of the country
d) None of the above
Q.96) Solution (d)
Explanation:
The objectives (or the functions) of the zonal councils, in detail, are as follows:
To achieve an emotional integration of the country.
To help in arresting the growth of acute state-consciousness, regionalism, linguism and
particularistic trends.
To help in removing the after-effects of separation in some cases so that the process of
reorganisation, integration and economic advancement may synchronise.
To enable the Centre and states to cooperate with each other in social and economic
matters and exchange ideas and experience in order to evolve uniform policies.
To cooperate with each other in the successful and speedy execution of major
development projects.
To secure some kind of political equilibrium between different regions of the country.

www.iasbaba.com

59

Polity- Subject wise compilation of 60 Day Plan- 2016

Q.97) The most productive sources of revenue in every federation are with the centre
while the most expensive heads of expenditure are with the states. This makes states
financially dependent on the center. Which of the following statements are correct about
Financial relations of Center and state?
1. In the matter of taxation, the constitution recognizes no concurrent jurisdiction. Hence
there is no subject who may be taxed both by the union and the state governments.
2. The share of states in tax revenue is decided by the NITI ayog.
3. The residuary power of taxation belongs to the centre.
Select the code from below:
a) 1 and 2
b) 2 and 3
c) 1 and 3
d) None of the above
Q.97) Solution (c)
The constitution grants the Union Parliament exclusive power to levy taxes on several
items.
The state legislatures enjoy similar power with regard to several other specified items. In
general, the Union Parliament levies taxes on items mentioned in the union list while the
state legislatures levy taxes on items mentioned in the state list.
The residuary power of taxation belongs to the centre. It means that the subjects which
have not been included either in the union or in the state list may be taxed only by the
union government.
In the matter of taxation, the constitution recognizes no concurrent jurisdiction. Hence
there is no subject who may be taxed both by the union and the state governments.
Q.98) Consider the following statements regarding Finance Commission:
1. Finance Commission is a statutory body made by Finance Commission Act 1948.
2. Finance Commission decides the subjects on which Center and States can levy taxes
respectively.
3. It determines the principles of governing the Grants in Aid to states.
4. Formation of NITI aayog has made Finance Commission redundant.
Which of the above statements are incorrect?
a) 1 only
b) 1 and 4 only
c) 1, 2 and 4 only
d) 2, 3 and 4 only

www.iasbaba.com

60

Polity- Subject wise compilation of 60 Day Plan- 2016


Q.98) Solution (c)
Finance Commission is a Constitutional Body.
Finance Commission does not decide the subjects on which Center and States can levy taxes
respectively. It only recommends the share of states in the revenue.
Since NITI Aayog does not interfere in any sort of revenue sharing, it has made the Finance
Commission more relevant which was earlier overshadowed by the Planning Commission.
Q.99) According to Article 360 of the Constitution, President can declare a Financial
Emergency. Consider the following statements regarding Financial Emergency:
1. The President can reduce the salaries of all government officials, excluding judges of the
a) Supreme Court and High Courts.
2. Fundamental Rights are suspended.
3. After the Money bills are passed by the Legislature of the State, they are reserved for
the final approval of the President.
Which of the above statements are incorrect?
a) 1 and 2
b) 2 and 3
c) 2 only
d) All of the above
Q.99) Solution (a)
In case of a financial emergency, the President can reduce the salaries of all government
officials, including judges of the Supreme Court and High Courts. All money bills passed by
the State legislatures are submitted to the President for his approval. He can direct the state
to observe certain principles (economy measures) relating to financial matters; but
fundamental rights cannot be suspended.
Q.100) Insolvency and Bankruptcy Code has been passed by the Government. Which of
the following are correct about the Insolvency Resolution Process:
1. The resolution process may be initiated by the debtor or creditors upon a default in
repayment.
2. The process will have to be completed within 360 days.
3. During the process, the creditors will decide to either restructure the companys debt, or
sell its assets to recover their outstanding dues.
4. The proceeds from sale will be distributed in an order of Priority.
Which of the above statements are correct?
a) 1,2 and 3
b) 2,3 and 4
c) 1,3 and 4

www.iasbaba.com

61

Polity- Subject wise compilation of 60 Day Plan- 2016


d) All of the above
Q.100) Solution (c)
The process has to be completed within 180 days. However, in certain cases it can be
extended up to 270 days.
Q.101) The Ministry of Women and Child Development released the draft Trafficking of
persons (Prevention, Protection and Rehabilitation) Bill, 2016. Which of the following are
the features of the draft bill?
1. Anti-trafficking bodies will be constituted at the district level only.
2. The District Anti-Trafficking body will be headed by a District Magistrate or District
a) Collector and will consists of five members.
3. A special Agency will be constituted for investigation of offences. A Sessions Court in
each district will be specified as a Special Court.
Select the code from the following:
a) 1 and 2
b) 2 and 3
c) 1 and 3
d) All of the above
Q.101) Solution (b)
Anti-trafficking bodies will be constituted at a district, state and Central Level.
District Committee will perform duties in relation to prevention trafficking and rescue
and rehabilitation of victims. It will be headed by a District Magistrate.
The anti-trafficking bodies at center and state will oversee the implementation of the
Act. The
State Anti-Trafficking Committee will be headed by the Chief Secretary and will have
nine members.
(Note: Read more about the bill.)
Q.102) In which of the following cases, The Parliament can make a law on a subject in the
State
List?
1. A proclamation of emergency is in force.
2. Two or more states make a request to the Parliament to make a law on a subject.
3. Rajya Sabha passes a resolution that such subjects have acquired national importance
Select the code from the following:
a) 1 only
b) 1 and 2

www.iasbaba.com

62

Polity- Subject wise compilation of 60 Day Plan- 2016


c) 1 and 3
d) 1,2 and 3
Q.102) Solution (d)
Direct Question. Self Explanatory.
Q.103) Consider the following statements:
1. Education was initially a subject under State list but later it was brought under
Central List.
2. To bring a subject from State list to Concurrent List a Constitutional Amendment is
required.
Which of the above statements are correct?
a) 1 only
b) 2 only
c) Both 1 and 2
d) Neither 1 nor 2
Q.103) Solution (b)
Education was brought from state list to concurrent list by 42nd Amendment.
*Note: In explanation it was wrongly given as 52 nd CAA, rather it should have been 42nd
CAA

Q.104) Consider the following statements regarding the Special Category Status awarded
to
states in India:
1. The Special Category Status to a state is awarded by National Development Council
(NDC).
2. Economic and infrastructural backwardness of a state is the only criterion to award
the special status.
3. Recently Telangana got the Special category status.
Which of the above statements are correct?
a) 1 only
b) 2 and 3
c) 1 and 3
d) All of the above
Q.104) Solution (a)
Some of the features required for special status are:
i. hilly and difficult terrain;
ii. low population density or sizeable share of tribal population;
iii. strategic location along borders with neighbouring countries;
iv. economic and infrastructural backwardness; and
v. non-viable nature of state finances.

www.iasbaba.com

63

Polity- Subject wise compilation of 60 Day Plan- 2016


Telangana has not been given any special status. However, Andhra Pradesh has requested
for the same. The decision has not been taken yet.
GoI has said that there is no further need of giving 'Special Category Status' but, the
previous status has not been officially abolished. So the question is valid as far as polity is
concerned.
Q.105) If the states fail to comply with the directives of the Central Government, which of
the following actions can be taken by the President?
1. He can send Paramilitary forces to state to assert compliance.
2. He can order fresh elections in state.
3. He can declare State emergency and assume the responsibility for the governance of the
state.
4. He can dissolve the Legislature and ask Governor to assume the responsibility of the
Governance of the state.
Select the code from following:
a) 1,2 and 3
b) 2,3 and 4
c) 3 only
d) 1,2 and 4
Q.105) Solution (c)
On the recommendation of Governor, the President can declare the State Emergency and
can take the responsibility of governance of state.
Q.106) Which of the following statement/s is/are correct regarding Constitutional
Amendment bill.
1. Prior permission of President is required before introducing the constitutional
amendment bill in parliament.
2. President must give his assent to the bill if duly passed by both houses.
Select the code from the following:
a) 1 only.
b) 2 only.
c) Both.
d) None
Q.106) Solution (b)
As per the procedure laid out by article 368 for amendment of the Constitution, an
amendment can be initiated only by the introduction of a Bill in either House of Parliament.
The Bill must then be passed in each House by a majority of the total membership of that

www.iasbaba.com

64

Polity- Subject wise compilation of 60 Day Plan- 2016


House and by a majority of not less than two-thirds of the members of that House present
and voting. There is no provision for a joint sitting in case of disagreement between the two
Houses. Total membership in this context has been defined to mean the total number of
members comprising the House irrespective of any vacancies or absentees on any account
vide Explanation to Rule
159 of the Rules of Procedure and Conduct of Business in Lok Sabha.
The Bill, passed by the required majority, is then presented to the President who shall give
his assent to the Bill.

Q.107) Under Article 352, the President can declare a national emergency when the
security of India or a part of it is threatened by
1) War
2) External aggression
3) Internal disturbance
Select the correct code:
a) 1 only
b) 2 only
c) 1 and 2 only
d) All of the above
Q.107) Solution (c)
Explanation:
Under Article 352, the President can declare a national emergency when the security of
India or a part of it is threatened by war or external aggression or armed rebellion (but
not on the ground of internal disturbance)
Q.108) Consider the following statements:
1) The Constitution declares Delhi as the seat of the Supreme Court. But, it also authorises
the chief justice of India to appoint other place or places as seat of the Supreme Court.
2) The CJI can take decision in this regard only with the approval of the President.
3) This means that Supreme Court can give direction either to the President or to the Chief
Justice to appoint any other place as a seat of the Supreme Court.
Which of the statements given above is/are incorrect?
a) 3 only
b) 1 and 2 only
c) 2 and 3 only
d) None of the above

www.iasbaba.com

65

Polity- Subject wise compilation of 60 Day Plan- 2016


Q.108) Solution (a)
Explanation:
The Constitution declares Delhi as the seat of the Supreme Court. But, it also authorises
the chief justice of India to appoint other place or places as seat of the Supreme Court.
He can take decision in this regard only with the approval of the President. This
provision is only optional and not compulsory.
This means that no court can give any direction either to the President or to the Chief
Justice to appoint any other place as a seat of the Supreme Court.
Q.109) Consider the following statements with regard to Parliamentary Approval of the
proclamation of Constitutional Emergency:
1) Once the President proclaims the Constitutional Emergency, it must be approved by
both the Houses of Parliament within one month from the date of its issue.
2) However, if the proclamation of the Constitutional Emergency is issued at a time
when the LS has been dissolved or the dissolution of the LS takes place during the
period of one month without approving the proclamation, then the proclamation
survives until 30 days from the first sitting of the Lok Sabha after its reconstitution,
provided the Rajya Sabha has in the meantime approved it.
Which of the statements given above is/are correct?
a) 1 only
b) 2 only
c) Both 1 and 2
d) Neither 1 nor 2
Q.109) Solution (d)
Explanation:
An Emergency due to the failure of the constitutional machinery in the states (Article
356). This is popularly known as Presidents Rule. It is also known by two other
namesState Emergency or Constitutional Emergency.
A proclamation imposing Presidents Rule must be approved by both the Houses of
Parliament within two months from the date of its issue.
However, if the proclamation of Presidents Rule is issued at a time when the Lok Sabha
has been dissolved or the dissolution of the Lok Sabha takes place during the period of
two months without approving the proclamation, then the proclamation survives until
30 days from the first sitting of the Lok Sabha after its reconstitution, provided the Rajya
Sabha approves it in the mean time.
*Note: When it is regard to proclamation of National Emergency, then the given statements
will be correct (i.e. approval by 1 month)
Doubt: thevagabond85

www.iasbaba.com

66

Polity- Subject wise compilation of 60 Day Plan- 2016


Q3. "...proclamation of Constitutional Emergency:" i used following q to narrow down to
A352 & 356 " is Emergency declared under A352 is an Un Constitutional Emergency? Why a
question reader would/should not consider emergency wrt to A352?
the phrase used must be either emergency imposed due to "failure of constitutional
machinery" or "emergency under A356" .......
Also : the endnote "*Note: When it is regard to proclamation of National Emergency, then
the given statements will be correct (i.e. approval by 1 month)" is slightly wrong. Through
out PART XVIII constitution uses the phrase "30 days" and never 1 month (although it uses
2m/6m) . :P :P
Clarification:
We agree vagabond, the phrase used in question was bit annoying or confusing. UPSC
doesn't frame question like this one particularly (our intention here was totally different, it
was to make everyone aware of the following concepts/facts) 1) In Constitution, "Proclamation of Emergency" equates to "National Emergency" only
2) President Rule is also known with namesState Emergency or Constitutional
Emergency
3) There is difference between Parliamentary approval and duration between President rule
and Proclamation of Emergency
4) And since the duration has been amended, our intention was to make you ppl know that
1 month for Nat Emergency and 2 months for Constitutional Emergency
So only with this kind of shocker in question can make u wake up to understand above
differences. Yes, it feels annoyed at first instance but later we have learnt some crucial
things which we usually skip.
And regarding your - phrase "30 days"
Read the statement carefully and again our note in the solution (you can notice that we are
not talking about that 30days wala provision)
Q.110) When there is a lack of quorum of the permanent judges to hold or continue any
session of the Supreme Court
1) The Chief Justice of India can appoint a judge of a High Court as an ad hoc judge of the
Supreme Court for a temporary period
2) The CJI can do so only after consultation with the chief justice of the High Court
concerned and with the previous consent of the president
Which of the statements given above is/are correct?
a) 1 only
b) 2 only

www.iasbaba.com

67

Polity- Subject wise compilation of 60 Day Plan- 2016


c) Both
d) None
Q.110) Solution (c)
Explanation:
Both the given statements are correct and self-explanatory.
Q.111) Which of the following statements is/are true in regard to the Judiciary?
1) India has a single system of courts, adopted from the Government of India Act of 1935
2) The Parliament is authorized to regulate the jurisdiction, powers, procedures and so on
of the Supreme Court
Which of the statements given above is/are correct?
a) 1 only
b) 2 only
c) Both
d) None
Q.111) Solution (c)
Explanation:
Both the given statements are correct and self-explanatory.
Articles 124 to 147 in Part V of the Constitution deal with the organisation,
independence, jurisdiction, powers, procedures and so on of the Supreme Court. The
Parliament is also authorized to regulate them.
Q.112) Consider the following statements:
1) Proclamation of a national emergency by President is not a discretionary power, as he
can proclaim only after receiving a written recommendation from the Prime Minister
2) In the Minerva Mills case, the Supreme Court held that the proclamation of a national
emergency can be challenged in a court on the ground of malafide or that the
declaration was based on wholly extraneous and irrelevant facts or is absurd or perverse
Which of the statements given above is/are correct?
a) 1 only
b) 2 only
c) Both 1 and 2
d) Neither 1 nor 2
Q.112) Solution (b)
Explanation:

www.iasbaba.com

68

Polity- Subject wise compilation of 60 Day Plan- 2016

The President can proclaim a national emergency only after receiving a written
recommendation from the cabinet.
This means that the emergency can be declared only on the concurrence of the cabinet
and not merely on the advice of the prime minister.
Statement 2 is correct and self-explanatory.

Q.113) Which of the following is not true in regard to the proclamation of Emergency?
a) The proclamation of Emergency must be approved by both the Houses of Parliament
within two months from the date of its issue
b) Every resolution approving the proclamation of emergency or its continuance must be
passed by either House of Parliament by a special majority
c) A proclamation of emergency may be revoked by the President at any time by a
subsequent proclamation. Such a proclamation does not require the parliamentary
approval
d) Further, the President must revoke a proclamation if the Lok Sabha passes a resolution
disapproving its continuation
Q.113) Solution (a)
Explanation:
The proclamation of Emergency must be approved by both the Houses of Parliament
within one month from the date of its issue
Q.114) A resolution of disapproval is different from a resolution approving the
continuation of a proclamation, because:
1) The first one is required to be passed by the Lok Sabha only, while the second one needs
to be passed by the both Houses of Parliament.
2) The first one is to be adopted by a simple majority only, while the second one needs to
be adopted by a special majority.
Which of the statements given above is/are correct?
a) 1 only
b) 2 only
c) Both 1 and 2
d) Neither 1 nor 2
Q.114) Solution (c)
Explanation:
Both the given statements are correct and self-explanatory
Q.115) Consider the following statements in regard to Supreme Courts Writ Jurisdiction:

www.iasbaba.com

69

Polity- Subject wise compilation of 60 Day Plan- 2016


1) The original jurisdiction of the Supreme Court with regard to federal disputes is different
from its original jurisdiction with regard to disputes relating to fundamental rights
2) In the first case, it is exclusive and in the second case, it is concurrent with high courts
jurisdiction
3) Moreover, the parties involved in the first case are units of the federation (Centre and
states) while the dispute in the second case is between a citizen and the Government
(Central or state)
Which of the statements given above is/are correct?
a) 1 only
b) 1 and 2 only
c) All of the above
d) None of the above
Q.115) Solution (c)
Explanation:
Both the given statements are correct and self-explanatory
Q.116) Consider the following statements in regard to the National Emergency:
1) During a national emergency, the executive power of the Centre extends to directing
any state regarding the manner in which its executive power is to be exercised.
2) Thus, the state governments are suspended and are brought under the complete control
of the Centre.
Which of the statements given above is/are correct?
a) 1 only
b) 2 only
c) Both
d) None
Q.116) Solution (a)
Explanation:
During a national emergency, the executive power of the Centre extends to directing
any state regarding the manner in which its executive power is to be exercised.
Thus, the state governments are brought under the complete control of the Centre,
though they are not suspended.
Q.117) The expression, Proclamation of Emergency as used in the Constitution refers to

a) National Emergency
b) State Emergency or Constitutional Emergency

www.iasbaba.com

70

Polity- Subject wise compilation of 60 Day Plan- 2016


c) Financial Emergency
d) All of the above
Q.117) Solution (a)
Explanation:
Article 352 Proclamation of Emergency The Constitution employs the expression
proclamation of emergency to denote the National Emergency only i.e. an emergency
due to war, external aggression or armed rebellion.
An Emergency due to the failure of the constitutional machinery in the states (Article
356). This is popularly known as Presidents Rule. It is also known by two other
namesState Emergency or constitutional Emergency. However, the Constitution
does not use the word emergency for this situation.
Financial Emergency is proclaimed under Article 360 due to a threat to the financial
stability or credit of India
Q.118) What happens when a proclamation of national emergency is in operation?
1) The President can issue ordinances on the state subjects also, if the Parliament is not
in session
2) The President can either reduce or cancel the transfer of finances from Centre to the
states
3) The six Fundamental Rights under Article 19 are automatically suspended
4) However, every such order [in statement (2) and (3)] of the President has to be laid
before both the Houses of Parliament
Select the correct answer using the code given below:
a) 1 and 2 only
b) 1, 2 and 3 only
c) 1, 3 and 4 only
d) All of the above
Note: Answer to this question is corrected. Read below for the changes and clarification.
Q.118) Solution (a)
Explanation:
Only 3rd and 4th statement is wrong.

While proclamation of national emergency is in operation, the President can either


reduce or cancel the transfer of finances from Centre to the states. However, every such
order of the President has to be laid before both the Houses of Parliament

Doubt: Jyotika
I think there is an error in Q.12. part C. It says that FR under Art 19 are automatically
suspended in case of emergency.

www.iasbaba.com

71

Polity- Subject wise compilation of 60 Day Plan- 2016


However FR under Art 19 are suspended automatically when emergency is proclaimed on
grounds of war or external aggression. However, if emergency is declared on grounds of
armed rebellion, FR under Art 19 are not automatically suspended, but can be suspended by
President by a separate proclamation.
Clarification:
Suspension of Fundamental Rights under Article 19
According to Article 358, when a proclamation of national emergency is made, the six
Fundamental Rights under Article 19 are automatically suspended. No separate order for
their suspension is required.
However, the 44th Amendment Act of 1978 restricted the scope of Article 358 in two ways.
Firstly, the six Fundamental Rights under Article 19 can be suspended only when the
National Emergency is declared on the ground of war or external aggression and not on the
ground of armed rebellion.
Q.119) Which of the following are the eligibility criteria for a person to be appointed as a
Supreme Court Judge?
1. The person must be a citizen of India
2. The person must have attained 35 years of age
3. A judge of one high court or more (continuously), for at least five years
4. An advocate in a High Court, for at least ten years
5. A distinguished jurist, in the opinion of the president
Select the code from below:
a) 3,4 and 5
b) 1,3,4 and 5
c) 1,2 and 3
d) All of the above
Q.119) Solution (b)
There isnt a minimum age limit to be appointed as a Supreme Court judge.
Q.120) Earlier the judges of the Supreme court were appointed by the President on the
Advice of the Union Cabinet. But the Scenario changed after the Second Judges Case of
1993. In the light of Second Judges Case, which of the following statements are correct?
1. Judges are appointed by the President on the recommendation of the Collegium
consisting of Chief Justice and senior most judges of the Supreme Court.
2. No Minister, or the executive collectively cannot suggest a name to president.
3. Collegium system was put into Constitution by 44th Amendment.
Select the code from below:

www.iasbaba.com

72

Polity- Subject wise compilation of 60 Day Plan- 2016


a) 1 and 2
b) 2 and 3
c) 1 and 3
d) All of the above
Q.120) Solution (a)

As per the Constitution, as held by the court in the Three Judges' Cases - (1982, 1993,
1998), a judge is appointed to the Supreme Court by the President of India on the
recommendation of the collegium a closed group of the Chief Justice of India, the four
most senior judges of the court and the senior-most judge hailing from the high court of
a prospective appointee. This has resulted in a Memorandum of Procedure being
followed, for the appointments.
Judges used to be appointed by him on the advice of the Union Cabinet. After 1993 (the
Second Judges' Case), no minister, or even the executive collectively, can suggest any
names to the President, who ultimately decides on appointing them from a list of names
recommended only by the collegium of the judiciary. Simultaneously, as held in that
judgment, the executive was given the power to reject a recommended name. However,
according to some, the executive has not been diligent in using this power to reject the
names of bad candidates recommended by the judiciary.
Collegium system is not given in the Constitution.

*Note: Statement (1) is modified and corrected


Q.121) Consider the following statements regarding Judiciary during British rule:
1. The Indian High Court Act was passed in 1861 and High Courts were established at
Calcutta, Bombay and Madras.
2. The judges in these High Courts were appointed by the Crown.
3. The Indian Penal Code was enacted in the year 1860.
Which of the above statements are correct?
a) 1 only
b) 1 and 2
c) 1 and 3
d) All of the above
Q.121) Solution (d)
The establishment of High Courts in Calcutta, Bombay and Madras: Under the
recommendations of commission appointed in 1853, The Government introduced important
reforms in the judicial system. Their recommendations were accepted and in 1861 the
Indian High Courts act authorized the establishment of High Courts in each of the following
towns; Calcutta, Bombay and Madras in place of old Supreme Court, Sadar Fojudari Adalat
and the Sadar Dewani adalat were abolished after having a age of 90 years.

www.iasbaba.com

73

Polity- Subject wise compilation of 60 Day Plan- 2016


"In 1866 a regulation, which was amended in later years, established a Chief Court for the
Punjab much on the same lines as the High Courts, though the judges were to be appointed
by the Governor General in Council and not as in the case of the High Courts by the Crown."
The Indian Penal Code was enacted in the year 1860 while Criminal Procedure Code in 1862.
Q.122) As per Article 143 the President has the power to address questions to the
Supreme Court, which he deems important for public welfare. Which of the following
statements are correct about advisory Jurisdiction of the Supreme Court?
1. The Advice given by the Supreme Court under Article 143 is not binding on the
President.
2. The Supreme Court may decline to give its opinion under Article 143(1) in cases it
does not consider proper or not amenable to such exercise.
3. References made by Supreme Court under Article 143 are binding on the inferior
Courts.
Select the code from below:
a) 1 and 2
b) 2 and 3
c) 1 and 3
d) All of the above
*Note: Statement (1) is corrected
Q.122) Solution (a)

Article 143 of the Indian Constitution confers upon the Supreme Court advisory
jurisdiction. The President may seek the opinion of the Supreme Court on any question
of law or fact of public importance on which he thinks it expedient to obtain such an
opinion. On such reference from the President, the Supreme Court, after giving it such
hearing as it deems fit, may report to the President its opinion thereon. The opinion is
only advisory, which the President is free to follow or not to follow. (Keshav Singhs
Case, AIR 1965 SC 745). However, even if the opinion given in the exercise of advisory
jurisdiction may not be binding, it is entitled to great weight.
The Supreme Court may decline to give its opinion under Article 143 in cases it does not
consider proper or not amenable to such exercise. It was, however, held by the Supreme
Court in M. Ismail Faruqui v. Union of India (AIR 1995 SC 605) that in that case, reasons
must be indicated.
It was also held by the Supreme Court that the references made under this Article are
not the law declared by the Supreme Court under Article 141 of the Constitution. So it
is not binding on inferior courts, even though have high persuasive value.

Q.123) Consider the following statements:


1. Family Courts are statutory provisions to secure speedy settlement of disputes related
to marriage and family affairs.
2. Family Courts are established by State Governments after consultation with High Courts.

www.iasbaba.com

74

Polity- Subject wise compilation of 60 Day Plan- 2016


Select the correct code from the following:
a) 1 only
b) 2 only
c) Both 1 and 2
d) Neither 1 nor 2
Q.123) Solution (c)
The Family Courts Act 1984 was enacted on 14 September 1984 to provide for the family
courts with a view to promoting conciliation in and secure speedy settlement of disputes
relating to marriage and family affairs. According to Section 2 (d) of the act, "Family Court"
means a family court established under section 3. Section 3 describes the establishment of
Family Courts and says that the State Government after consultation with the High Court
and by notification shall establish a Family Court for every area of the state consisting of a
city or town whose population exceeds ten lakhs and for other areas in the state as it may
deem necessary. Family courts are subordinate to the High Court, which has power to
transfer the case from one family court to the other.
Q.18) Consider the following statements:
1. e-Courts is a Mission Mode Project for justice delivery by ICT enablement of Courts.
2. According to the project, all the courts including taluk courts will get computerised.
3. The project also includes producing witnesses through video conferencing.
Which of the above statements are correct?
a) 1 only
b) 1 and 3
c) 2 and 3
d) All of the above
Q.124) Solution (d)
The e-Courts project was conceptualized on the basis of the National Policy and Action
Plan for Implementation of information and communication technology (ICT) in the Indian
Judiciary 2005 submitted by e-Committee (Supreme Court of India), with a vision to
transform the Indian Judiciary by ICT enablement of Courts.
The project envisages:
To provide efficient & time-bound citizen centric service delivery.
To develop, install & implement decision support systems in courts.
To automate the processes to provide transparency of Information access to its
stakeholders.
To enhance judicial productivity both qualitatively & quantitatively, to make the justice
delivery system affordable, accessible, cost effective & transparent.

www.iasbaba.com

75

Polity- Subject wise compilation of 60 Day Plan- 2016


The E-courts project was established in the year 2005. According to the project, all the
courts including taluk courts will get computerised. As per the project in 2008, all the
District courts were initialised under the project. In 2010, all the District court were
computerised. The entry of back log case has started. The IT department had one system
officer and two system assistants in each court. They initiated the services in the Supreme
Court in June 2011. The case lists and the judgements of most district courts are available in
http://lobis.nic.in. in http://judis.nic.in is used to connect all High Courts and Supreme Court
judgements and cause list. These websites are updated daily by a technical team. Now the
establishment work is going on taluk courts.
The project also includes producing witnesses through video conferencing. Filing cases,
proceedings, and all other details will be in computers. Each district court contains 1 system
officer and 2 system assistants. This technical manpower is involved in training the staff,
updating web sites.
Q.19) Consider the following statements regarding the writ jurisdiction of High Courts:
1. The writ jurisdiction of High Court is mentioned under Article 32 of the Indian
Constitution.
2. The Supreme Court has wider powers as compared to High Courts in issuing writs.
Which of the above statements are incorrect?
a) 1 only
b) 2 only
c) Both 1 and 2
d) Neither 1 nor 2
Q.124) Solution (c)
Both 1 and 2 are incorrect.
The Writ Jurisdiction of Supreme Court is mentioned under Article 32 of the Indian
Constitution, while the Writ Jurisdiction of High Courts is mentioned under Article 226 of the
Indian Constitution.
The High Courts have wider powers as to compare to Supreme Court in issuing writs. The
Supreme Court can issue writ only in case of violation of any of the fundamental rights
contained in Part-III of the constitution, while the High Courts can issue writs not only in
case of violation of fundamental rights but also in case of violation of any legal rights of the
citizens provided that a writ is a proper remedy in such cases, according to well-established
principles.
Q.125) Two or more states can have a common High Court. A common High Court for two
or more states and Union Territory may be established by:
a) The President
b) The Parliament by law

www.iasbaba.com

76

Polity- Subject wise compilation of 60 Day Plan- 2016


c) The State Governments of the Respective states
d) The Governors of the respective states
Q.125) Solution (b)
Self explanatory. Direct question.
Q.126) Consider the following in regard to the powers and actions taken by the National
Human Rights Commission:
1) NHRC has wide-ranging powers to investigate, recommend prosecutions, and award
compensations for human rights violations.
2) NHRC can take any coercive measures and take action against persons or authorities
who do not follow the guidelines laid down by it.
Which of the statements given above is/are correct?
a) 1 only
b) 2 only
c) Both 1 and 2
d) Neither 1 nor 2
Q.126) Solution (a)

The functions of the commission are mainly recommendatory in nature.


It has no power to punish the violators of human rights, nor to award any relief including
monetary relief to the victim. Notably, its recommendations are not binding on the
concerned government or authority. But, it should be informed about the action taken
on its recommendations within one month.

Doubts:
Sir, Q. 1, Statement 1 says that NHRC can award compensations. This is wrong. As per
Human rights (amendment) act, 2006, (as given in Laxmikant), NHRC is empowered to
RECOMMEND AWARD OF COMPENSATION. Plz clarify.. Isn't RECOMMENDING award of
compensation different from AWARDING the same??
Clarifications:
Question
was
framed
from
the
Hindu
article
http://www.thehindu.com/opinion/op-ed/comment-on-justice-sathasivam-running-fornhrc-office/article7282984.ece (Read 1st para)
It says NHRC has wide-ranging powers to investigate, recommend prosecutions, and
award compensations for human rights violations.
However, Laxmikanth mentions NHRC is empowered to RECOMMEND AWARD OF
COMPENSATION
So may be a typing mistake by the Hindu It should have been NHRC has wide-ranging
powers to investigate, to recommend prosecutions and award compensations for human
rights violations.
www.iasbaba.com

77

Polity- Subject wise compilation of 60 Day Plan- 2016


So let us consider question as
Q.126) Consider the following in regard to the powers and actions taken by the National
Human Rights Commission:
1) NHRC has wide-ranging powers to investigate, to recommend prosecutions and
award compensations for human rights violations.
2) NHRC can take any coercive measures and take action against persons or authorities
who do not follow the guidelines laid down by it.
Which of the statements given above is/are correct?
a) 1 only
b) 2 only
c) Both 1 and 2
d) Neither 1 nor 2
Q.126) Solution (a)
Q.127) Which among the following statement(s) is/are correct about Cabinet Secretariat?
1) The Cabinet Secretariat functions directly under the Prime Minister.
2) The administrative head of the Secretariat is the Cabinet Secretary who is also the exofficio Chairman of the Civil Services Board.
3) The business allotted to Cabinet Secretariat is Secretarial assistance to Cabinet and
Cabinet Committees; and Rules of Business.
Choose the correct answer from the codes given below:
a) 1 only
b) 1 and 2 only
c) 1 and 3 only
d) All of the above
Q.127) Solution (d)
The Cabinet Secretariat functions directly under the Prime Minister. The administrative
head of the Secretariat is the Cabinet Secretary who is also the ex-officio Chairman of
the Civil Services Board.
The business allotted to Cabinet Secretariat is (i) Secretarial assistance to Cabinet and
Cabinet Committees; and (ii) Rules of Business.
The Cabinet Secretariat is responsible for the administration of the Government of India
(Transaction of Business) Rules, 1961 and the Government of India (Allocation of
Business) Rules, 1961, facilitating smooth transaction of business in
ministries/departments of the Government by ensuring adherence to these rules. It
assists in decision-making by ensuring inter-ministerial coordination, ironing out

www.iasbaba.com

78

Polity- Subject wise compilation of 60 Day Plan- 2016

differences amongst Ministries/Departments and evolving consensus through the


instrumentality of the standing and ad hoc Committees of Secretaries.
The Cabinet Secretariat ensures that the President, the Vice-President and Ministers are
kept informed of the major activities of all ministries/departments by means of monthly
summary of their activities. Management and coordinating activities of various
ministries in such a situation is also one of the functions of the Cabinet Secretariat.
The Cabinet Secretariat is a useful mechanism by the departments for promoting interministerial coordination since the Cabinet Secretary is also the head of the Civil Services.
The Secretaries keep the Cabinet Secretary informed of developments from time to
time. The Transaction of Business Rules also require them to keep the Cabinet Secretary
informed specially if there are any departures from these rules.

Q.128) Consider the following statements about National Investigation Agency:


1) The NIA was created by an Act of the Parliament following Mumbai Terror Attack of
November 2008
2) The NIA deals with offences under only eight laws, which also includes among them the
Atomic Energy Act, 1962
Which of the statements given above is/are correct?
a) 1 only
b) 2 only
c) Both 1 and 2
d) Neither 1 nor 2
Q.29) Solution (c)
Both the given statements are correct and self-explanatory.
Q.130) Which among the following is true regarding the composition of NHRC?
1) The NHRC, set up under the Protection of Human Rights Act, 1993, consists of nine
members.
2) Four are ex-office appointments serving Chairpersons of the National
Commissions for Minorities, Scheduled Castes, Scheduled Tribes and Women.
3) Two are persons who have done work in the area of human rights.
4) Three are from the judiciary: a sitting or retired judge of the Supreme Court; a Chief
Justice of a High Court; and, the most important of all, a former Chief Justice of India
(CJI) who heads the Commission.
Choose the appropriate code:
a) 1 only
b) 1, 2 and 3 only
c) 1, 2 and 4 only

www.iasbaba.com

79

Polity- Subject wise compilation of 60 Day Plan- 2016


d) All of the above
Q.130) Solution (d)

All the given statements are correct and direct factual question (self-explanatory)

Doubts:
SIr, Questions no: 4 , option 4 ) Shouldn't the statement be: a sitting or retried Chief Justice
of High Court?
Clarifications:
Again the question was framed from same The Hindu article http://www.thehindu.com/opinion/op-ed/comment-on-justice-sathasivam-running-fornhrc-office/article7282984.ece (Read 2nd para)
However,
Laxmikanth
and
NHRC
website
(http://nhrc.nic.in/documents/Publications/TheProtectionofHumanRightsAct1993_Eng.p
df) says this
The commission (NHRC) is a multi-member body consisting of a chairman and four
members.
1) The chairman should be a retired chief justice of India
2) A serving or retired judge of the Supreme Court
3) A serving or retired chief justice of a high court and
4) Two persons having knowledge or practical experience with respect to human rights.
In addition to these full-time members, the commission also has four ex-officio members
the chairmen of the National Commission for Minorities, the National Commission for SCs,
the National Commission for STs and the National Commission for Women.
Let us consider the lines from the above Hindu article as - Three are from the judiciary: a
sitting or retired judge of the Supreme Court; a sitting or retired Chief Justice of a High
Court; and, the most important of all, a former Chief Justice of India (CJI) who heads the
Commission.
Question should be read asQ.130) Which among the following is true regarding the composition of NHRC?
1) The NHRC, set up under the Protection of Human Rights Act, 1993, consists of nine
members.
2) Four are ex-office appointments serving Chairpersons of the National
Commissions for Minorities, Scheduled Castes, Scheduled Tribes and Women.
3) Two are persons who have done work in the area of human rights.
4) Three are from the judiciary: a sitting or retired judge of the Supreme Court; a sitting
or retired Chief Justice of a High Court; and, the most important of all, a former Chief
Justice of India (CJI) who heads the Commission.

www.iasbaba.com

80

Polity- Subject wise compilation of 60 Day Plan- 2016


Choose the appropriate code:
a) 1 only
b) 1, 2 and 3 only
c) 1, 2 and 4 only
d) All of the above
Q.130) Solution (d)
Q.131) The National Human Rights Commissions public face and guiding force is the
Chairperson. Consider the following statements in regard to Chairperson:
1) The Chairperson is appointed by the president on the recommendations of a sixmember committee.
2) Only a former CJI or a puisne judge of the Supreme Court are considered for the post.
3) The chairman hold office for a term of five years or until they attain the age of 70 years.
Which of the statements given above is/are correct?
a) All of the above
b) 1 and 3 only
c) 1 and 2 only
d) 3 only
Q.131) Solution (b)
The Chairperson is appointed by the president on the recommendations of a sixmember committee consisting of the prime minister as its head, the Speaker of the Lok
Sabha, the Deputy Chairman of the Rajya Sabha, leaders of the Opposition in both the
Houses of Parliament and the Central home minister.
Only a former CJI and not even a puisne judge of the Supreme Court is considered for
the post. Hence statement (2) is wrong.
Q.132) Consider the following in regard to the Chief Information Commissioner and
Information Commissioner:
1) The Chief Information Commissioner and an Information Commissioner hold office for a
term of 5 years or until they attain the age of 70 years, whichever is earlier.
2) They are not eligible for reappointment.
3) The Commission can order inquiry into any matter if there are reasonable grounds (suomoto power).
4) The Commission is empowered to utilise the services of any officer or investigation
agency of the Central government or any state government for the purpose.
Which of the statements given above is/are correct?
a) All of the above

www.iasbaba.com

81

Polity- Subject wise compilation of 60 Day Plan- 2016


b) 2 and 3 only
c) 1 and 2 only
d) 2, 3 and 4 only
Q.132) Solution (b)
The Chief Information Commissioner and an Information Commissioner hold office for a
term of 5 years or until they attain the age of 65 years, whichever is earlier.
The commission is not empowered to utilise the services of any officer or investigation
agency of the Central government or any state government for the purpose.
Q.133) Which among the following statement is/are true about Higher Education in India?
1) Education falls under the Concurrent List of the Constitution
2) The mandate of determining standards of higher education and research lies with the
centre, as this falls under the Union List
3) States have powers to incorporate, regulate and wind up universities as a subject under
the State List
Select the appropriate option:
a) All the above
b) 1 and 2 only
c) 1 and 3 only
d) 1 only
Q.133) Solution (a)
All of the above given statements are correct self explanatory.
Q.134) Consider the statements related to Fast Track Courts (FTCs):
1) FTCs were to be established by the state governments in consultation with the Supreme
Court
2) The judges for these FTCs were appointed by promoting members from amongst the
eligible judicial officers on ad-hoc basis only.
3) Under the scheme average of five FTCs were to be established in each district of the
country.
Choose the correct codes
a) 1 and 2
b) 2 only
c) 3 only
d) All of the above
Q.134) Solution (c)

www.iasbaba.com

82

Polity- Subject wise compilation of 60 Day Plan- 2016

FTCs were to be established by the state governments in consultation with the


respective High Courts.
An average of five FTCs was to be established in each district of the country.
The judges for these FTCs were appointed on an ad-hoc basis. The judges were selected
by the High Courts of the respective states.
There are primarily three sources of recruitment.
i. First, by promoting members from amongst the eligible judicial officers;
ii. Second, by appointing retired High Court judges and
iii. Third, from amongst members of the Bar of the respective state.
Source:http://www.thehindu.com/news/national/tamil-nadu/fast-track-court-judgesappointed-on-ad-hoc-basis-seek-pension/article7669652.ece
Q.135) Which among the following is/are not true about Zonal Councils?
1) The Zonal Councils are neither statutory nor constitutional bodies.
2) There are 5 zonal councils (Northern, Central, Eastern, Western and Southern)
3) The Prime Minister is the common chairman of the five zonal councils.
4) Each chief minister acts as a vice-chairman of the council by rotation, holding office for a
period of one year at a time.
Choose the appropriate codes from below options
a) 1 only
b) 1 and 3
c) 1 and 4
d) 3 and 4
Q.135) Solution (b)
The Zonal Councils are the statutory (and not the constitutional) bodies. They are
established by an Act of the Parliament, that is, States Reorganisation Act of 1956. The
act divided the country into five zones (Northern, Central, Eastern, Western and
Southern) and provided a zonal council for each zone.
The home minister of Central government is the common chairman of the five zonal
councils. Each chief minister acts as a vice-chairman of the council by rotation, holding
office for a period of one year at a time.
The zonal councils aim at promoting cooperation and coordination between states,
union territories and the Centre. They discuss and make recommendations regarding
matters like economic and social planning, linguistic minorities, border disputes, interstate transport, and so on. They are only deliberative and advisory bodies.
Q.136) Which of the following bodies does not/do not find mention in the Constitution?
1) National Development Council
www.iasbaba.com

83

Polity- Subject wise compilation of 60 Day Plan- 2016


2) Planning Commission
3) Zonal Councils
Select the correct answer using the codes given below.
a) 1 and 2 only
b) 2 only
c) 1 and 3 only
d) 1, 2 and 3
Q.136) Solution (d)
Factual Question All the given bodies does not find mention in the Constitution.
Q.137) Who of the following are part of NITI Aayog but were not a part of Planning
Commission?
1. Prime Minister
2. Chief Ministers of All states
3. Lieutenant Governors of Union Territories
4. Chief Secretaries of All States
Select the code from below:
a) 1 and 2
b) 2 and 3
c) 2,3 and 4
d) All of the above
Q.137) Solution (b)
Prime Minister was the Ex officio Chairman of the Planning commission.
Chief Secretaries are not a part of NITI Aayog.
Q.138) Which of the following actions can be taken by National Human Rights Commission
(NHRC) with respect to alleged inhuman conditions of relief camps for riots victims in a
state in India:
1. Taking suo motu cognizance, NHRC may order independent probe to inquire into
violation of human rights but only within one year of the incident.
2. NHRC may order the state government to make payment of compensation or damages
to the victims.
Which of the above statements are correct?
a) 1 only
b) 2 only
c) Both 1 and 2

www.iasbaba.com

84

Polity- Subject wise compilation of 60 Day Plan- 2016


d) Neither 1 nor 2
Q.138) Solution (a)
NHRC may recommend (not Order) the state Government to make payment of
compensation or damages to the victims.
Q.139) Consider the following statements regarding Central Information Commission
(CIC):
1. It was constituted through an Official Gazette Notification under the provisions of the
Right to Information Act (2005).
2. It entertains complaints and appeals pertaining to offices, financial institutions, public
sector undertakings, etc., under the Central Government, State Government and the
Union Territories.
Which of the above statements are incorrect?
a) 1 only
b) 2 only
c) Both 1 and 2
d) Neither 1 nor 2
Q.139) Solution (b)
It entertains complaints and appeals pertaining to offices, financial institutions, public
sector undertakings, etc., under the Central Government and the Union Territories.
Q.140) Consider the following:
Central Body/Commission
1. UPSC0
2. Central Vigilance Commission
3. Central Information Commission
4. Staff Selection Commission

Ministry
Ministry of Personnel
Ministry of Personnel
Ministry of Personnel
Ministry of Personnel

Which of the above are correctly matched?


a) 1 and 4
b) 2 and 3
c) 1,2 and 4
d) All of the above
Q.140) Solution (d)
All the above bodies falls under the ministry of Personnel.
Q.141) Consider the following statements regarding Central Vigilance Commission:

www.iasbaba.com

85

Polity- Subject wise compilation of 60 Day Plan- 2016


1. The Central Vigilance Commissioner and vigilance commissioners are appointed by the
president by warrant under his hand and seal.
2. They hold office for a term of six years or until they attain the age of sixty five years,
whichever is earlier.
3. They are not eligible for further employment under the Central or a state government.
Which of the above statements are correct?
a) 1 and 2
b) 2 and 3
c) 1 and 3
d) 3 only
Q.141) Solution (c)
They hold office for a term of four years or until they attain the age of sixty five years,
whichever is earlier.
Q.142) Consider the following statements
1. National green tribunal is enacted under Article 21 of Indian constitution.
2. The tribunal is guided by principles of natural justice and not code of civil procedure,
1908
Select the correct option
a) 1 only
b) 2 only
c) Both
d) None
Q.2) Solution (c)
Doubt: it asks "National green tribunal is enacted under Article 21 of Indian constitution."
The tribunal is made under NGT ACT 2010 and not article 21. Honorable SC only said that
right to clean environment is part of right to life. It only draws "inspiration" from article 21
and not "enacted" under it. Kindly clarify!
Clarification: Very true. Correct answer should be:
Q.142) Solution (b)
Q.143) The final interpreter of the provisions of the Constitution of India within the House
is
a) Attorney General
b) President

www.iasbaba.com

86

Polity- Subject wise compilation of 60 Day Plan- 2016


c) Speaker
d) Supreme Court
Q.143) Solution (c)
Explanation:
Speaker is the final interpreter of the provisions of (a) the Constitution of India, (b) the
Rules of Procedure and Conduct of Business of Lok Sabha, and (c) the parliamentary
precedents, wit
Q.144) Consider the following statements in regard to representation of UTs in the second
chamber:
1) The Constitution has empowered the Parliament to prescribe the manner of choosing
the representatives of the UTs to the second chamber
2) The Parliament has enacted the Union Territories (Indirect Election to the Second
Chamber) Act, 1965, by which the representatives of the UTs are chosen
3) The representatives of each UT in the second chamber are indirectly elected by
members of an electoral college specially constituted for the purpose
4) This election is held in accordance with the system of proportional representation by
means of the single transferable vote
Which of the statements given above is/are correct?
a) 2 and 3 only
b) 3 and 4 only
c) 1, 3 and 4 only
d) All of the above
Q.144) Solution (c)
Explanation:
Representation of Union Territories in Rajya Sabha
Article 80(5) of Indian Constitution states that The representatives of the Union
Territories in the council of States shall be chosen in such manner as Parliament may by
law prescribe
Accordingly, the Parliament has introduced PART IVA (MANNER OF FILLING SEATS IN
THECOUNCIL OF STATES TO BE FILLED BY REPRESENTATIVES OF UNION TERRITORIES) in
THE REPRESENTATION OF THE PEOPLE ACT, 1950
The representatives of each union territory in the Rajya Sabha are indirectly elected by
members of an electoral college specially constituted for the purpose. This election is
also held in accordance with the system of proportional representation by means of the
single transferable vote.

www.iasbaba.com

87

Polity- Subject wise compilation of 60 Day Plan- 2016

Out of the seven union territories, only two (Delhi and Puducherry) have representation
in Rajya Sabha. The populations of other five union territories are too small to have any
representative in the Rajya Sabha.

Doubts: baba ji in question 12 : statement 1 should be correct . Article 80 (5) The


representatives of the Union territories in the council of states shall be chosen in such a
manner as parliament may by law prescribe. (Thank you @vivek gupta)
Clarification: This answer has been corrected for a factual error. Please refer to the newly
added explanation above.
Q.145) Consider the following statements:
1) Under the Rules of Rajya Sabha, the Chairman nominates from amongst the members a
panel of vice-chairpersons
2) Any of them can preside over the House when the office of the Speaker or the Deputy
Speaker is vacant
Which of the statements given above is/are incorrect?
a) 1 only
b) 2 only
c) Both 1 and 2
d) Neither 1 nor 2
Q.145) Solution (b)
Explanation:
Under the Rules of Rajya Sabha, the Chairman nominates from amongst the members a
panel of vice Any one of them can preside over the House in the absence of the Chairman or the
Deputy Chairman.
It must be emphasized here that a member of the panel of vice-chairpersons cannot
preside over the House, when the office of the Chairman or the Deputy Chairman is

During such time, the Chairmans duties are to be performed by such member of the
House as the president may appoint for the purpose. The elections are held, as soon as
possible, to fill the vacant posts.

Clarification: This question has been corrected for a factual error. Please refer second
statement.
*Although there was no error in its previous format of the question.

www.iasbaba.com

88

Polity- Subject wise compilation of 60 Day Plan- 2016


Q.146) Which among the given statements is/are true in regard to the Speaker of Lok
Sabha?
1) He is the guardian of powers and privileges of the members, the House as a whole and
its committees
2) President enables the House to elect the new Speaker
3) Parliamentary Affairs Committee, which looks after the progress of government business
in the Parliament, is chaired by the Speaker
Select the appropriate code
a) 1 only
b) 1 and 2 only
c) 2 only
d) All of the above
Q.146) Solution (a)
Explanation:
Speaker is the guardian of powers and privileges of the members, the House as a whole
and its committees.
The main duty of Speaker Pro Tem is to administer oath to the new members. He also
enables the House to elect the new Speaker.
Parliamentary Affairs Committee, which looks after the progress of government business
in the Parliament, is chaired by the Home Minister (not Speaker).
Doubts: Thanks for such sharp questions but Sir
In Q14, 2nd statement, as President appoints speaker pro tem, in a way he enables the
house to elect new speaker (by appointing speaker pro tem.)Please clarify.
Clarification: The date of election of the Speaker is fixed by the President. The President
also appoints a member of the Lok Sabha as the Speaker Pro Tem.
The Speaker Pro Tem has all the powers of the Speaker. He presides over the first sitting of
the newly-elected Lok Sabha. His main duty is to administer oath to the new members. He
enables the House to elect the new Speaker.
(Presidents job is finished fixing the election date and appointing a Speaker Pro Tem. The
entire remaining task is carried out by Speaker Pro Tem and enables the House to elect the
new Speaker)
In the Lok Sabha, both the Speaker and the Deputy Speaker are elected from among its
members by a simple majority of members present and voting in the House. As such, no
specific qualifications are prescribed for being elected as the Speaker.

www.iasbaba.com

89

Polity- Subject wise compilation of 60 Day Plan- 2016

The Constitution only requires that Speaker should be a member of the House. But,
Parliamentary conventions and procedure play a major while deciding the holder of the
office of the Speaker. (Here, role of Speaker Pro Tem is important)
Usually, a member belonging to the ruling party is elected as the Speaker. A healthy
convention, however, has evolved over the years whereby the ruling party nominates its
candidate after informal consultations with the Leaders of other Parties and Groups in the
House. This convention ensures that once elected, the Speaker enjoys the respect of all
sections of the House. There are also instances when members not belonging to the ruling
party or coalition were elected to the office of the Speaker. (Again here, Speaker Pro Tem
role comes)
Q.147) From the following devices of Parliamentary Proceedings, select only the Indian
innovated Parliamentary procedures:
1) Zero hour
2) Short notice question
3) Calling attention motion
4) No-day- yet-named motion
Choose appropriate option:
a) 1 and 3
b) 2 and 3
c) 1, 2 and 4
d) 2, 3 and 4
Q.147) Solution (a)
Explanation:
Question Hour: The first hour of every parliamentary sitting is slotted for this. During
this time, the members ask questions and the ministers usually give answers. The

A short notice question is one that is asked by giving a notice of less than ten days. It is
answered orally.
Unlike the question hour, the zero hour is not mentioned in the Rules of Procedure.
Thus it is an informal device available to the members of the Parliament to raise matters
without any prior notice.
The zero hour starts immediately after the question hour and lasts until the agenda for
the day (ie, regular business of the House) is taken up. In other words, the time gap
between the question hour and the agenda is known as zero hour. It is an Indian
innovation in the field of parliamentary procedures and has been in existence since
1962.

www.iasbaba.com

90

Polity- Subject wise compilation of 60 Day Plan- 2016

Calling Attention Motion: It is introduced in the Parliament by a member to call the


attention of a minister to a matter of urgent public importance, and to seek an
authoritative statement from him on that matter. Like the zero hour, it is also an Indian
innovation in the parliamentary procedure and has been in existence since 1954.
However, unlike the zero hour, it is mentioned in the Rules of Procedure.

Q.148) Consider the following statements in regard to Money Bill and Financial Bill:
1) Money bills and financial bills can be introduced only on the recommendation of the
President
2) Financial Bills are governed by the same legislative procedure which is applicable to an
ordinary bill
3) The only special feature of Financial Bills is that it cannot be passed by either House of
Parliament unless the President has recommended to that House the consideration of
the bill
Which of the statements given above is/are correct?
a) 1 and 3 only
b) 1 and 2 only
c) 2 only
d) 3 only
Q.148) Solution (d)
Explanation:
Money Bill and Financial Bill (I) can be introduced only on the recommendation of the
President, but recommendation of the President is not necessary for the introduction of
Financial bill (II). Therefore, not all financial bills require prior recommendation of the
President to be introduced.
*Very Important: Only some of the financial bills are governed by the same legislative
procedure which is applicable to an ordinary bill. Since Finance bills are of three kinds
Money bills, Financial bills (I) and (II), all bills are not governed by the same legislative
procedure which is applicable to an ordinary bill.
The only special feature of Financial Bill (II) is that it cannot be passed by either House of
Parliament unless the President has recommended to that House the consideration of
the bill (Financial Bill (I) has no such special feature)
https://indiankanoon.org/doc/1704729/
Clarification: This question has been corrected for a factual error. Please refer second
statement and answer.
Q.149) Which among the following statements is/are not true in regard to Speaker and
Deputy Speaker of Lok Sabha?

www.iasbaba.com

91

Polity- Subject wise compilation of 60 Day Plan- 2016


1) The institutions of Speaker and Deputy Speaker originated in India under the provisions
of the Government of India Act of 1935
2) President administers the oath of the Speaker and Speaker administers the oath of
Deputy Speaker and remaining members of the LS
3) Speaker acts as strictly a non-party man in India, he resigns from the membership of his
party on his election to the exalted office
Select the appropriate code
a) 1 and 2 only
b) 2 and 3 only
c) 2 only
d) All of the above
Q.149) Solution (d)
Explanation:
The institutions of Speaker and Deputy Speaker originated in India in 1921 under the
provisions of the Government of India Act of 1919 (MontagueChelmsford Reforms). At
that time, the Speaker and the Deputy Speaker were called the President and Deputy

The Speaker and the Deputy Speaker, while assuming their offices, do not make and
subscribe any separate oath or affirmation.
In Britain, the Speaker is strictly a non-party man. There is a convention that the Speaker
has to resign from his party and remain politically neutral. This healthy convention is not
fully established in India where the Speaker does not resign from the membership of his
party on his election to the exalted office.

Q.150) Which among the following is/are correct in regard to Motions associated with
Parliamentary functions?
1) No-Confidence Motion and Censure motion can be moved against the entire council of
ministers
2) Calling Attention Motion is introduced in the Parliament to draw attention of the House
to a definite matter of urgent public importance
3) All resolutions come in the category of substantive motion
Select the appropriate code
a) 1 and 3 only
b) 2 and 3 only
c) 2 only
d) All of the above
Q.150) Solution (a)

www.iasbaba.com

92

Polity- Subject wise compilation of 60 Day Plan- 2016


Explanation:
Censure motion can be moved against an individual minister or a group of ministers or
the entire council of ministers. No-Confidence Motion can be moved against the entire
council of ministers only.
It is Adjournment Motion (not Calling Attention Motion), which is introduced in the
Parliament to draw attention of the House to a definite matter of urgent public
importance.
Calling Attention Motion is introduced in the Parliament by a member to call the
attention of a minister (not the House) to a matter of urgent public importance, and to
seek an authoritative statement from him on that matter.
All resolutions come in the category of substantive motions, that is to say, every
resolution is a particular type of motion.
Q.151) Match List I with List II and select the correct answer using the code given below
the Lists:
List I
(Parliamentary Committees)
A.
Public
Accounts 1)
Committee
B. Estimates Committee
2)
C. Standing committee
D.
Committee
Empowerment
of Women

3)
on 4)

List II
(Consists of)
22 members (15
and 7 from RS)
30 members (All
only)
31 members (21
and 10 from RS)
30 members (20
and 10 from RS)

from LS
from LS
from LS
from LS

A-B-C-D
a) 1-2-4-3
b) 2-1-4-3
c) 2-1-3-4
d) 1-2-3-4
Q.151) Solution (d)
Explanation:
Self Explanatory
Note: This questions intention is to make you aware of the Committees of the
Parliament and its composition (as UPSC has started asking questions on these recently)
Q.152) Consider the following statements with regard to Parliamentary Privileges:

www.iasbaba.com

93

Polity- Subject wise compilation of 60 Day Plan- 2016


1) Without these privileges, the Parliament can neither maintain their authority, dignity
and honor nor can protect their members from any obstruction in the discharge of their
parliamentary responsibilities
2) The Constitution has extended Parliamentary privileges to the Parliament, including the
attorney general of India
Which of the statements given above is/are correct?
a) 1 only
b) 2 only
c) Both 1 and 2
d) Neither 1 nor 2
Q.152) Solution (d)
Explanation:
Parliamentary privileges are special rights, immunities and exemptions enjoyed by the
two Houses of Parliament, their committees and their members. They are necessary in
order to secure the independence and effectiveness of their actions.
Without these privileges, the Houses (not the Parliament as it includes President of India
and both the houses) can neither maintain their authority, dignity and honor nor can
protect their members from any obstruction in the discharge of their parliamentary
responsibilities.
The Constitution has also extended the parliamentary privileges to those persons who
are entitled to speak and take part in the proceedings of a House of Parliament or any of
its committees. These include the attorney general of India and Union ministers.
It must be clarified here that the parliamentary privileges do not extend to the president
who is also an integral part of the Parliament.
Note: As Parliament consists of President, both the statements are incorrect.
Note: Source for all these questions is Laxmikanth, Parliament Chapter
Q.153) Consider the following regarding Public Accounts Committee
1. The chairman of the committee must belong to the ruling party
2. The function of the committee is to examine the annual audit reports of the Comptroller
and Auditor General of India (CAG), which are laid before the Parliament by the Speaker
3. It is a non-advisory and non-executive body
Select the correct code
a) 1 and 2
b) Only 2
c) Only 3
d) None

www.iasbaba.com

94

Polity- Subject wise compilation of 60 Day Plan- 2016


Q.153) Solution (d)
Until 1966-67, the chairman of the committee belonged to the ruling party. However, since
1967 a convention has developed whereby the chairman of the committee is selected
invariably from the Opposition. The function of the committee is to examine the annual
audit reports of the Comptroller and Auditor General of India (CAG), which are laid before
the Parliament by the President. The CAG submits three audit reports to the President,
namely, audit report on appropriation accounts, audit report on finance accounts and audit
report on public undertakings.
However, the effectiveness of the role of the committee is limited by the following:
a) It is not concerned with the questions of policy in broader sense.
b) It conducts a post-mortem examination of accounts (showing the expenditure already
incurred).
c) It cannot intervene in the matters of day-to-day administration.
d) Its recommendations are advisory and not binding on the ministries.
e) It is not vested with the power of disallowance of expenditures by the departments.
f) It is not an executive body and hence, cannot issue an order. Only the Parliament can
take a final decision on its findings
Q.154) Consider the following w.r.t Ethics Committee of Lok Sabha and Rajya Sabha
1. Ethics Committee has an Ad Hoc status in Lok Sabha whereas a Permanent Standing
Committee Status in Rajya Sabha
2. Both the houses explicitly provide for a Register of Members Interest, where MPs have
to declare their interest in certain categories
3. Lok Sabhas Ethics Committee acts both on complaints as well as takes up issues suo
motu, Rajya Sabhas committee acts only on complaints made either by any member of
the public or any other member of the House
Select the incorrect statement/s
a) 1 and 2 Only
b) 1, 2 and 3
c) 2 and 3
d) Neither of them
Q.154) Solution (b)
Lok Sabha has a new permanent Standing Committee on Ethics which came into force on
12th August, 2015. Until now, the ethics committee in Lok Sabha has been an ad hoc one.
Rajya Sabha has explicitly provided for a Register of Members Interest, where MPs have to
declare their interest in 5 categories: remunerative directorship, remunerated activity,
majority shareholding, paid consultancy and professional engagement. In addition to that,

www.iasbaba.com

95

Polity- Subject wise compilation of 60 Day Plan- 2016


members are required to declare any financial interest on an issue that is being debated in
the House or under consideration by any other standing committee and hence refrain from
taking part to avoid conflict of interest.
Lok Sabha does not maintain such a registry of members interests and apart from disclosing
their assets and liabilities, MPs are not obliged to declare other financial interests that might
be in direct or indirect conflict with their role as public servants.
Another significant point of difference between the two Houses is that while Rajya Sabhas
Ethics Committee acts both on complaints as well as takes up issues suo motu, Lok Sabhas
committee acts only on complaints made either by any member of the public or any other
member of the House.
Q.155) Panchayats receive funds from which of the following resources?
1) Funds released by State Governments on recommendations of Central Finance
Commission
2) Funds for implementation of centrally sponsored schemes
3) Local body grants as recommended by Central Finance Commission
Select the correct answer using the codes given below:
a) 1 only
b) 2 and 3 only
c) 3 only
d) 1, 2 and 3
Q.155) Solution (d)
Doubts: Q 6: In statement 1st it should be state finance commission ???
Clarification:
Yes, it is true that State Finance Commission governs the distribution between the state and the panchayats of the net proceeds of the taxes,
duties, tolls and fees levied by the state.
determination of taxes, duties, tolls and fees that may be assigned to the
panchayats.
grants-in-aid to the panchayats from the consolidated fund of the state.
However, there is also a provision that the Central Finance Commission can recommend
certain measures to the State governments how to augment the consolidated fund of a
state and release additional funds or resources to Panchayats (apart from those already
recommended by the finance commission of the state).
Read Finance Commission part in Panchayat Chapter (Indian Polity by Laxmikanth)
Q.156) Consider the following statements:

www.iasbaba.com

96

Polity- Subject wise compilation of 60 Day Plan- 2016


1) Collective responsibility is the bedrock principle of parliamentary government
2) The ministers are collectively responsible to the Lok Sabha
3) The principle of collective responsibility implies that the Parliament can remove the
ministry from office by passing a vote of no confidence.
Choose the correct codes
a) 1 and 2
b) 2 and 3
c) 2 only
d) All of the above
Q.156) Solution (a)

Note: Question [statement (2)] has been corrected.


Explanation:
Collective responsibility is the bedrock principle of parliamentary government. The
ministers are collectively responsible to the Parliament in general and to the Lok Sabha
in particular (Article 75). They act as a team, and swim and sink together.
The principle of collective responsibility implies that the Lok Sabha can remove the
ministry (i.e., council of ministers headed by the prime minister) from office by passing a
vote of no confidence.

Q.157) Consider the following statements about the appointment of CVC:


1) He is appointed by the President
2) The recommendation of the nominee of the post comes from the Council of Ministers
3) Appointment of CVC cannot be challenged in court
Choose the correct answer using the codes below
a) 1 and 2 only
b) 2 and 3 only
c) 1 and 3 only
d) 1 only
Q.157) Solution (d)
Explanation:
The CVC is a multi-member body consisting of a Central Vigilance Commissioner
(chairperson) and not more than two vigilance commissioners.

They are appointed by the president by warrant under his hand and seal on the
recommendation of a three-member committee consisting of the prime minister as its head,
the Union minister of home affairs and the Leader of the Opposition in the Lok Sabha.
www.iasbaba.com

97

Polity- Subject wise compilation of 60 Day Plan- 2016


They hold office for a term of four years or until they attain the age of sixty five years,
whichever is earlier.
After their tenure, they are not eligible for further employment under the Central or a state
government.
Q.158) Consider the following statements regarding Special Officer for Linguistic Minorities:
1. The Original Constitution did not have any provision for Special Officer for Linguistic
Minorities.
2. The commissioner for Linguistic Minorities has his head office at Allahabad.
Which of the above statements are correct?
a) 1 only
b) 2 only
c) Both 1 and 2
d) Neither 1 nor 2
Q.158) Solution (c)
Originally,the Constitution of India did not make any provision with respect to the Special
Officer for Linguistic Minorities1. Later, the States Reorganisation Commission (1953-55)
made a recommendation in this regard. Accordingly, the Seventh Constitutional
Amendment Act of 1956 inserted a new Article 350-B in Part XVII of the Constitution.

In pursuance of the provision of Article 350-B of the Constitution, the office of the Special
Officer for Linguistic Minorities was created in 1957. He is designated as the Commissioner
for Linguistic Minorities.
The Commissioner has his headquarters at Allahabad (Uttar Pradesh). He has three regional
offices at Belgaum (Karnataka), Chennai (Tamil Nadu) and Kolkata (West Bengal). Each is
headed by an Assistant Commissioner.
Q.159) Consider the following statements related to organisation of Panchayats
1) A Gram Sabha consists of all the adults i.e. voters living in the area of a Gram Panchayat.
2) Gram Sabha is an executive committee but not a legal body.
3) Gram Panchayat is the village assembly of Gram Sabha.
Choose the correct codes
a) 1 only
b) 2 only
c) 3 only
d) None of the above
Q.159) Solution (a)

www.iasbaba.com

98

Polity- Subject wise compilation of 60 Day Plan- 2016

Explanation:
The Village Panchayat or Gram Panchayat is the executive committee of Gram Sabha.

A Gram Sabha or Village Assembly consists of all the adults i.e. voters (persons above the
age of 18 years) living in the area of a Gram Panchayat i.e., village or a group of small
villages.
The Gram Sabha has now been recognized as a legal body.

Q.160) Consider the following statements:


1) Collective responsibility is the bedrock principle of parliamentary government
2) The ministers are collectively responsible to the Parliament
3) The principle of collective responsibility implies that the Parliament can remove the ministry
from office by passing a vote of no confidence.
Choose the correct codes
a) 1 and 2
b) 2 and 3
c) 2 only
d) All of the above
Q.160) Solution (a)
Explanation:
Collective responsibility is the bedrock principle of parliamentary government. The ministers are
collectively responsible to the Parliament in general and to the Lok Sabha in particular (Article
75). They act as a team, and swim and sink together.
The principle of collective responsibility implies that the Lok Sabha can remove the ministry (i.e.,
council of ministers headed by the prime minister) from office by passing a vote of no
confidence.
Q.161) With regard to Finance Commission, consider the following statements
1) It is a quasi-judicial body.
2) It is constituted by the President every fifth year or even earlier.
3) It is required to make recommendations to the Parliament on the distribution of the net
proceeds of taxes to be shared between the Centre and the states, and the allocation between
the states, the respective shares of such proceeds.
4) The Constitution envisages the Finance Commission as the balancing wheel of fiscal
federalism in India.
Choose the correct codes from below options:
a) 1, 2 and 3
b) 1, 2 and 4

www.iasbaba.com

99

Polity- Subject wise compilation of 60 Day Plan- 2016


c) 2, 3 and 4
d) 1 and 4
Q.161) Solution (b)
Explanation:
Article 280 provides for a Finance Commission as a quasi-judicial body. It is constituted by the
President every fifth year or even earlier. It is required to make recommendations to the
President on the following matters:
The distribution of the net proceeds of taxes to be shared between the Centre and the
states, and the allocation between the states, the respective shares of such proceeds.
The principles which should govern the grants-in- aid to the states by the Centre (i.e.,
out of the Consolidated Fund of India).
The measures needed to augment the Consolidated fund of a state to supplement the
resources of the panchayats and the municipalities in the state on the basis of the
recommendations made by the State Finance Commission.
Any other matter referred to it by the President in the interests of sound finance.
The Constitution envisages the Finance Commission as the balancing wheel of fiscal federalism
in India.
Q.162) Panchayats receive funds from which of the following resources?
1) Funds released by State Governments on recommendations of Central Finance Commission
2) Funds for implementation of centrally sponsored schemes
3) Local body grants as recommended by Central Finance Commission
Select the correct answer using the codes given below:
a) 1 only
b) 2 and 3 only
c) 3 only
d) 1, 2 and 3
Q.162) Solution (d)
Explanation:
Self-explanatory Factual question
Doubts: Q 6: In statement 1st it should be state finance commission ???

Clarification:
Yes, it is true that State Finance Commission governs the distribution between the state and the panchayats of the net proceeds of the taxes,
duties, tolls and fees levied by the state.
determination of taxes, duties, tolls and fees that may be assigned to the
panchayats.
grants-in-aid to the panchayats from the consolidated fund of the state.

www.iasbaba.com

100

Polity- Subject wise compilation of 60 Day Plan- 2016


However, there is also a provision that the Central Finance Commission can recommend
certain measures to the State governments how to augment the consolidated fund of a
state and release additional funds or resources to Panchayats (apart from those already
recommended by the finance commission of the state).
Read Finance Commission part in Panchayat Chapter (Indian Polity by Laxmikanth)

Q.163) Sources of Income of Gram Panchayats are:


1) taxes on property, land, goods and cattle
2) grants-in- aid from the State government only
3) land revenue collected by the Panchayats
4) tax on extraction of minerals
Choose the correct codes from below options:
a) 1 only
b) 1, 2 and 3
c) 2 only
d) All of the above
Q.163) Solution (a)
Explanation:
The sources are as follows:

taxes on property, land, goods and cattle; rent collected for facilities like Barat Ghar or
any other property of panchayat;
various types of fines collected from the offenders;
grants-in- aid from the State government and Union government;
a part of the land revenue collected by the State government (not collected by the
Panchayats) and is given to the Panchayats;
collecting tax on extraction of minerals is an exclusive power of State government
donations collected from the villagers for some common cause.

Q.164) Right to Livelihood as part of the Right to Life was established in


a) A.D.M. Jabalpur Vs S. Shukla 1976
b) Maneka Gandhi versus Union of India 1978
c) Olga Tellis vs Bombay Municipal Corporation 1985

www.iasbaba.com

101

Polity- Subject wise compilation of 60 Day Plan- 2016


d) Indira Sawhney Vs Union of India 1992
Q.164) Solution (c)
Explanation:
In A.D.M. Jabalpur Vs S. Shukla 1976, the Supreme Court declared the right to move
court under Articles 14, 21 and 22 would remain suspended during the Emergency.

The Maneka Gandhi Vs Union of India 1978 case caused a huge uproar over the
definition of freedom of speech. The court ruled that the procedure must be fair and the
law must not violate other fundamental rights.

In Indira Sawhney Vs Union of India 1992, the Supreme Court upheld the
implementation of recommendations made by the Mandal Commission. It also defined
the "creamy layer" criteria and reiterated that the quota could not exceed 50 per cent.

In Olga Tellis vs Bombay Municipal Corporation 1985, the SC observed, "the sweep of
the Right to Life, conferred by Article 21 is wide and far reaching. 'Life' means something
more than mere animal existence. It does not mean merely that life cannot be
extinguished or taken away as, for example, by the imposition and execution of the
death sentence, except according to procedure established by law. That is but one
aspect of the Right to Life. An equally important facet of that right is the right to
livelihood because no person can live without the means of living, that is, the means of
livelihood."

Link: http://www.thehindu.com/todays-paper/tp-national/tp-andhrapradesh/indiscriminateland-acquisition-scuttling-right-to livelihood/article4913615.ece

Q.165) Which among the following is/are the powers and responsibilities that are delegated
to panchayats at the appropriate level?
1) Preparation of the economic development plan and social justice plan.
2) Implementation of schemes for economic development and social justice in relation to 29
subjects given in the Eleventh Schedule of the Constitution.
3) To levy and collect appropriate taxes, duties, tolls and fees.
Choose the appropriate answer:
a) 1 only
b) 2 only
c) 1 and 2 only
d) All of the above
Q.165) Solution (d)

www.iasbaba.com

102

Polity- Subject wise compilation of 60 Day Plan- 2016


Explanation:
Self-Explanatory All the given statements are correct.
Q.166) Consider the following statements:
1) The state governments are under the constitutional obligation to adopt the new Panchayati
Raj system.
2) The compulsory provisions of the act have to be included in the state laws creating the new
Panchayati Raj system.
3) The voluntary provisions may be included at the discretion of the states.
Which of the above given statement(s) is/are incorrect in regard to Panchayati Raj System of
India?
a) 3 only
b) 1 and 2 only
c) 1 only
d) None
Q.165) Solution (d)
Explanation:
Factual question All the given statements are correct.
Q.166) The Constitution (Seventy-Third Amendment) Act. 1992, which aims at promoting the
Panchayati Raj Institutions in the country, provides for which of the following?
1. Constitution of District Planning Committees.
2. State Election Commissions to conduct all panchayat elections.
3. Establishment of state Finance Commissions.
Select the correct answer using the codes given below:
a) 1 only
b) 1 and 2 only
c) 2 and 3 only
d) 1, 2 and 3

district pl.com-ART 243ZD

Q.166) Solution (c)


District planning committees in India: District Planning Committee (DPC) is the committee
created as per article 243ZD of the Constitution of India at the district level for planning at the
district and below. Article 243ZD was added under 74th Amendment Act. 1992.
Q.167) Which of the following committees were appointed by the government specifically for
Panchayti Raj?
1. Balwantrai Mehta Committee
2. Ashok Mehta Committee
3. G V K Rao Committee

www.iasbaba.com

103

Polity- Subject wise compilation of 60 Day Plan- 2016


4. L M Singhvi Committee
Select the code from below:
a) All of the above
b) 1 and 2
c) 2 and 4
d) 3 and 4

Q.167) Solution (c)


(Repeated question from our test series to emphasise on clarity)
Balwant Rai Mehta Committee: In January 1957, the Government of India appointed a
committee to examine the working of the Community Development Programme (1952) and the
National Extension Service (1953) and to suggest measures for their better working.
Ashok Mehta Committee:nIn December 1977, the Janata Government appointed a committee
on panchayati raj institutions under the chairmanship of Ashok Mehta.
G V K Rao Committee: The Committee on Administrative Arrangement for Rural Development
and Poverty Alleviation Programmes under the chairmanship of G.V.K. Rao was appointed by
the Planning Commission in 1985.
L M Singhvi Committee:
In 1986, Rajiv Gandhi government appointed a committee on Revitalisation of Panchayati Raj
Institutions for Democracy and Development under the chairmanship of L M Singhvi.

Q.168) Which of the following statements are correct recommendations of the Ashok Mehta
Committee?
1. The three-tier system of panchayati raj should be replaced by the two-tier system.
2. The panchayati raj institutions should have compulsory powers of taxation to mobilise their
own financial resourses.
3. Political parties should not participate in panchayati elections.
Select the code from the following:
a) 1 and 2
b) 2 and 3
c) 1 and 3
d) All of the above
Q.168) Solution (a)

The three-tier system of panchayati raj should be replaced by the two-tier system, that
is, zila parishad at the district level, and below it, the mandal panchayat consisting of a
group of villages with a total population of 15,000 to 20,000.

www.iasbaba.com

104

Polity- Subject wise compilation of 60 Day Plan- 2016

A district should be the first point for decentralisation under popular supervision below
the state level.
Zila parishad should be the executive body and made responsible for planning at the
district level.
There should be an official participation of political parties at all levels of panchayat
elections.
The panchayati raj institutions should have compulsory powers of taxation to mobilise
their own financial resourses.

Q.169) Who of the following personalities came up with the concept of Gram Swaraj?
a) Madan Mohan Malviya
b) J P Narayan
c) Vallab bhai Patel
d) Mahatma Gandi
Q.169) Solution (d)
Self explanatory.
Q.170) Consider the following statements regarding State Finance Commission:
1. It is appointed by the State Government in every five years.
2. It determines development requirement for the State Government for formulating State Five
Year plan.
3. It determines the budgetary requirements of different departments of state.
4. It determines pattern of distribution of states tax revenue between the state government
and local bodies (both rural and urban) and the pattern of grant-in-aid to local bodies.
Which of the above statements are correct?
a) 1,2 and 3
b) 4 only
c) 1 and 4
d) All of the above
Q.170) Solution (c)

State Finance Commissions has been established in the various states of India so that they
can help in improving the financial condition of the various local bodies such as Panchayati
raj institutions and municipal bodies that are there in the states.
State Finance Commissions have been set up in the various states of the country according
to the guidelines that have been laid down in the Constitution of India, Article 243 (I).
According to this article, the governor of the state shall set up the Finance Commission
within the period of one year that begins with the seventy- third amendment act of the
Indian Constitution, 1992 and after that at the end of every five years. The Finance
www.iasbaba.com

105

Polity- Subject wise compilation of 60 Day Plan- 2016


Commissions in the States usually comprises of the chairman, member secretary, and other
members. State Finance Commissions receive grants from the Finance Commission that is
set up by the central government.
Q.171) According to 73rd Constitutional Amendment Act, the minimum age for contesting
elections to the Panchayati Raj Institutions should be
a) 18 years
b) 30 years
c) 21 years
d) 25 years
Q.171) Solution (c)
Self-explanatory
Q.172) Consider the following statements regarding the Civil Society Organisations:
1. They are sovereign bodies.
2. They are voluntary, non-profit making bodies.
3. They work in close proximity with people.
4. They create awareness; educate and sensitise people on common issues and act as catalysts
of social change.
Which of the above statements are correct?
a) All of the above
b) 1,2 and 3
c) 1,3 and 4
d) 2,3 and 4
Q.172) Solution (d)
Civil Society Organisations are not Sovereign bodies.
Q.173) Consider the following statements:
1. A cantonment board is established for municipal administration for civilian population in the
cantonment area.
2. A cantonment board is created as well as administered by the Central government.
3. It works under the administrative control of the defence ministry of the Central government.
Which of the above statements are correct?
a) 1 and 3
b) 1 and 2
c) 2 and 3
d) All of the above

www.iasbaba.com

106

Polity- Subject wise compilation of 60 Day Plan- 2016

Q.173) Solution (d)


A cantonment board is established for municipal administration for civilian population in the
cantonment area7. It is set up under the provisions of the Cantonments Act of 2006a
legislation enacted by the Central government. It works under the administrative control of
the defence ministry of the Central government. Thus, unlike the above four types of urban
local bodies, which are created and administered by the state government, a cantonment
board is created as well as administered by the Central government.

Q.174) Under which of the following circumstances can President remove the Chairman or any
other member of UPSC?
1. If he is adjudged an insolvent (that is, has gone bankrupt).
2. If he engages, during his term of office, in any paid employment outside the duties of his
Office.
3. If he is, in the opinion of the Supreme Court, unfit to continue in office by reason of infirmity
of mind or body.
Select the code from following:
a) 1 and 2
b) 2 and 3
c) 1 and 3
d) All of the above
Q.174) Solution (a)
The Chairman or the member of UPSC can be removed if he is, in the opinion of the President,
unfit to continue in office by reason of infirmity of mind or body.
Q.175) Which of the following statements correctly defines the term Gerrymandering?
a) Manipulate the boundaries of (an electoral constituency) so as to favour one party or class.
b) Ballot capturing or illegal voting.
c) Stopping a certain category of people from voting so that one specific party can be benefitted.
d) None of the above
Q.175) Solution (a)

www.iasbaba.com

107

S-ar putea să vă placă și